PN 140 Final Exam NCLEX Practice Questions

अब Quizwiz के साथ अपने होमवर्क और परीक्षाओं को एस करें!

In planning for the discharge of a client with a cognitive disorder, it is important to assess the client's caregiver support system. Which aspects are the most crucial to assess? Select all that apply. 1. Availability of resources for caregiver support. 2. Ability to provide the level of care and supervision needed by the client. 3. Willingness to transport the client to medical and psychiatric services. 4. Interest in engaging the cognitively disordered family member in reminiscence and games. 5. Willingness to install door alarms and make other safety changes. 6. Understanding the client's abilities and limitations.

1, 2, 3, 5, 6. It is important for a caregiver to have support for herself as well as be able to provide adequate safety, supervision, and medical care to the client. The caregiver must also have realistic expectations of the client, given his abilities and limitations. Reminiscing and engaging the client in games is desirable but not crucial to care.

A client has been diagnosed with hyperthyroidism. Which signs and symptoms may indicate thyroid storm, a complication of this disorder? Select all that apply. 1.Fever 2.Nausea 3.Lethargy 4.Tremors 5.Confusion 6.Bradycardia

1, 2, 4, 5 Thyroid storm is an acute and life-threatening condition that occurs in a client with uncontrollable hyperthyroidism. Symptoms of thyroid storm include elevated temperature (fever), nausea, and tremors. In addition, as the condition progresses, the client becomes confused. The client is restless and anxious and experiences tachycardia.

A client with a history of renal calculi formation is being discharged after surgery to remove the calculus. What instructions should the nurse include in the client's discharge teaching plan? 1. Increase daily fluid intake to at least 3 to 4 L. 2. Strain urine at home regularly. 3. Eliminate dairy products from the diet. 4. Follow measures to alkalinize the urine.

1. A high daily fluid intake is essential for all clients who are at risk for calculi formation because it prevents urinary stasis and concentration, which can cause crystallization. Depending on the composition of the stone, the client also may be instructed to institute specific dietary measures aimed at preventing stone formation. Clients may need to limit purine, calcium, or oxalate. Urine may need to be either alkaline or acid. There is no need to strain urine regularly.

The client in the early stage of Alzheimer's disease and his adult son attend an appointment at the community mental health center. While conversing with the nurse, the son states, "I'm tired of hearing about how things were 30 years ago. Why does Dad always talk about the past?" The nurse should tell the son: 1. "Your dad lost his short-term memory, but he still has his long-term memory." 2. "You need to be more accepting of your dad's behavior." 3. "I want you to understand your dad's level of anxiety." 4. "Telling your dad that you are tired of hearing about the past will help him stop."

1. The son's statements regarding his father's recalling past events is typical for family members of clients in the early stage of Alzheimer's disease, when recent memory is impaired. Telling the son to be more accepting is critical and not an attempt to educate. Understanding the client's level of anxiety is unrelated to the memory loss of Alzheimer's disease. The client cannot stop reminiscing at will.

The nurse is assessing the client diagnosed with congestive heart failure. Which laboratory data would indicate that the client is in severe congestive heart failure? 1. An elevated B-type natriuretic peptide (BNP) 2. An elevated creatine kinase (CK-MB) 3. A positive D-Dimer 4. A positive ventilation/perfusion (V/Q) scan

1. BNP is a specific diagnostic test. Levels higher than normal indicate congestive heart failure, with the higher the number the more severe the CHF

The nurse is preparing a client diagnosed with amyotrophic lateral sclerosis (ALS) for discharge. The nurse realizes that interventions for the following nursing diagnosis should be stressed for this client: 1. Ineffective Breathing Pattern related to neuromuscular dysfunction. 2. Impaired Urinary Elimination related to spastic or flaccid bladder. 3. Alteration in Vision acuity related to ocular muscle involvement. 4. Disturbed Thought Processes related to cognitive decline.

1. Ineffective Breathing Pattern related to neuromuscular dysfunction. Rationale: ALS affects the neuromuscular function; the client is at risk for respiratory dysfunction as a result of this disease process. Impaired urinary elimination is a result of a spastic or flaccid bladder resulting from MS. Visual acuity is impaired in clients diagnosed with MG. Disturbed thought processes is seen in AD clients.

The nurse is reviewing the lab results for a pt with cirrhosis and notes that the ammonia level is elevated. Which diet does the nurse anticipate to be prescribed for this pt? 1. Low-protein 2. High-protein 3. Moderate-fat 4.High-carb

1. Low-protein diet Protein provided by the diet is transported to the liver via the portal vein. The liver breaks down protein, which results in the formation of ammonia.

The nurse is preparing an educational program on amyotrophic lateral sclerosis (ALS). The nurse recognizes that this information is most appropriately presented at a: 1. Men's "50 or older" bowling league banquet. 2. Mother-and-daughter softball league season kickoff brunch. 3. "Singles over 60" wellness health fair. 4. A teenage men's hockey team annual fund raising event.

1. Men's "50 or older" bowling league banquet. Rationale: The onset of ALS typically occurs between 40 and 60 years of age, affecting men more often than women. While it can affect younger and older people of either gender, the 50-or-older male group would be the target population of the available options.

The client newly diagnosed with multiple sclerosis (MS) states, "I don't understand how I got multiple sclerosis. Is it genetic?" On which statement should the nurse base the response? 1. Genetics may play a role in susceptibility to MS, but the disease may be caused by a virus. 2. There is no evidence suggesting there is any chromosomal involvement in developing MS. 3. Multiple sclerosis is caused by a recessive gene, so both parents had to have the gene for the client to get MS. 4. Multiple sclerosis is caused by an autosomal dominant gene on the Y chromosome,so only fathers can pass it on.

1. The exact cause of MS is not known,but there is a theory stating a slow virus is partially responsible. A failure of apart of the immune system may also beat fault. A genetic predisposition involving chromosomes 2, 3, 7, 11, 17, 19, and X may be involved. 2. There is some evidence supporting a genetic component involved in developing MS. 3. A specific gene has not been identified to know if the gene is recessive or dominant. 4. The X chromosome, not the Y chromosome,may be involved.

A client with acute kidney injury is receiving a fluid challenge of 500 mL of normal saline over 1 hour. With a drop factor of 20 drops/mL, how many drops per minute does the nurse infuse?

167 drops/min 20 gtt × 500 mL = 10,000/60 min = 167 drops/min

A 69-year-old client is admitted and diagnosed with delirium. Later in the day, he tries to get out of the locked unit. He yells, "Unlock this door. I've got to go see my doctor. I just can't miss my monthly Friday appointment." Which of the following responses by the nurse is most appropriate? 1. "Please come away from the door. I'll show you your room." 2. "It's Tuesday and you are in the hospital. I'm Anne, a nurse." 3. "The door is locked to keep you from getting lost." 4. "I want you to come eat your lunch before you go the doctor."

2. Loss of orientation, especially for time and place, is common in delirium. The nurse should orient the client by telling him the time, date, place, and who the client is with. Taking the client to his room and telling him why the door is locked does not address his disorientation. Telling the client to eat before going to the doctor reinforces his disorientation.

Which of the following is essential when caring for a client who is experiencing delirium? 1. Controlling behavioral symptoms with low-dose psychotropics. 2. Identifying the underlying causative condition or illness. 3. Manipulating the environment to increase orientation. 4. Decreasing or discontinuing all previously prescribed medications.

2. The most critical aspect when caring for the client with delirium is to institute measures to correct the underlying causative condition or illness. Controlling behavioral symptoms with low-dose psychotropics, manipulating the environment, and decreasing or discontinuing all medications may be dangerous to the client's health.

A nurse on the Geropsychiatric unit receives a call from the son of a recently discharged client. He reports that his father just got a prescription for memantine (Namenda) to take "on top of his donepezil (Aricept)." The son then asks, "Why does he have to take extra medicines?" The nurse should tell the son: 1. "Maybe the Aricept alone isn't improving his dementia fast enough or well enough." 2. "Namenda and Aricept are commonly used together to slow the progression of dementia." 3. "Namenda is more effective than Aricept. Your father will be tapered off the Aricept." 4. "Aricept has a short half-life and Namenda has a long half-life. They work well together."

2. The two medicines are commonly given together. Neither medicine will improve dementia, but may slow the progression. Neither medicine is more effective than the other; they act differently in the brain. Both medicines have a half-life of 60 or more hours.

At what time of day should the nurse encourage a client with Parkinson's disease to schedule the most demanding physical activities to minimize the effects of hypokinesia? 1. Early in the morning, when the client's energy level is high. 2. To coincide with the peak action of drug therapy. 3. Immediately after a rest period. 4. When family members will be available.

2. Demanding physical activity should be performed during the peak action of drug therapy. Clients should be encouraged to maintain independence in self-care activities to the greatest extent possible. Although some clients may have more energy in the morning or after rest, tremors are managed with drug therapy.

Which signs/symptoms should make the nurse suspect the client is experiencing a thyroid storm? 1. Obstipation and hypoactive bowel sounds. 2. Hyperpyrexia and extreme tachycardia. 3. Hypotension and bradycardia. 4. Decreased respirations and hypoxia.

2. Hyperpyrexia (high fever) and heart rate above 130 beats per minute are signs of thyroid storm, a severely exaggerated hyperthyroidism. 1. These are signs of myxedema (hypothyroidism) coma. Obstipation is extreme constipation. 3. Decreased blood pressure and slow heartrate are signs of myxedema coma. 4. These are signs/symptoms of myxedema coma.

The home care nurse is visiting a client with a diagnosis of Parkinson's disease. The client is taking benztropine mesylate (Cogentin) orally daily. The nurse provides information to the spouse regarding the side effects of this medication and should tell the spouse to report which side effect if it occurs? 1. Shuffling gait 2. Inability to urinate 3. Decreased appetite 4. Irregular bowel movements

2. Inability to urinate Rationale: Benztropine mesylate is an anticholinergic, which causes urinary retention as a side effect. The nurse would instruct the client or spouse about the need to monitor for difficulty with urinating, a distended abdomen, infrequent voiding in small amounts, and overflow incontinence. Options 1, 3, and 4 are unrelated to the use of this medication.

A client with Parkinson's disease is prescribed levodopa (l-dopa) therapy. Improvement in which of the following indicates effective therapy? 1. Mood. 2. Muscle rigidity. 3. Appetite. 4. Alertness.

2. Levodopa is prescribed to decrease severe muscle rigidity. Levodopa does not improve mood, appetite, or alertness in a client with Parkinson's disease.

The nurse observes that a client's upper arm tremors disappear as he unbuttons his shirt. Which statement best guides the nurse's analysis of this observation about the client's tremors? 1. The tremors are probably psychological and can be controlled at will. 2. The tremors sometimes disappear with purposeful and voluntary movements. 3. The tremors disappear when the client's attention is diverted by some activity. 4. There is no explanation for the observation; it is probably a chance occurrence.

2. Voluntary and purposeful movements often temporarily decrease or stop the tremors associated with Parkinson's disease. In some clients, however, tremors may increase with voluntary effort. Tremors associated with Parkinson's disease are not psychogenic but are related to an imbalance between dopamine and acetylcholine. Tremors cannot be reduced by distracting the client.

The nurse is preparing a client with a new diagnosis of hypothyroidism for discharge. The nurse determines that the client understands discharge instructions if the client states that which symptoms are associated with this diagnosis? Select all that apply. 1.Tremors 2.Weight loss 3.Feeling cold 4.Loss of body hair 5.Persistent lethargy 6.Puffiness of the face

3, 4, 5, 6

The physician orders risperidone (Risperdal) for a client with Alzheimer's disease. The nurse anticipates administering this medication to help decrease which of the following behaviors? 1. Sleep disturbances. 2. Concomitant depression. 3. Agitation and assaultiveness. 4. Confusion and withdrawa

3. Antipsychotics are most effective with agitation and assaultiveness. Antipsychotics have little effect on sleep disturbances, concomitant depression, or confusion and withdrawal.

The nurse is caring for clients on a medical unit. After the shift report, which client should be assessed first? 1. the 34-year old client who is quadriplegic and cannot move his arms. 2. the elderly client diagnosed with a CVA who is weak on the right side. 3. The 78 year old client with pressure ulcers who has a temperature of 102.3 4. The young adult who is unhappy with the care that was provided last shift.

3. The 78 year old client with pressure ulcers who has a temperature of 102.3

The home health nurse is visiting a client with a diagnosis of multiple sclerosis. The client has been taking oxybutynin (Ditropan XL). The nurse evaluates the effectiveness of the medication by asking the client which assessment question? 1. "Are you consistently fatigued?" 2. "Are you having muscle spasms?" 3. "Are you getting up at night to urinate?" 4. "Are you having normal bowel movements?"

3. "Are you getting up at night to urinate?" Rationale: Oxybutynin is an antispasmodic used to relieve symptoms of urinary urgency, frequency, nocturia, and incontinence in clients with uninhibited or reflex neurogenic bladder. Expected effects include improved urinary control and decreased urinary frequency, incontinence, and nocturia. Options 1, 2, and 4 are unrelated to the use of this medication.

The client diagnosed with PD is being discharged on carbidopa/levodopa (Sinemet), .an antiparkinsonian drug. Which statement is the scientific rationale for combining these medications? 1. There will be fewer side effects with this combination than with carbidopa alone. 2. Dopamine D requires the presence of both of these medications to work. 3. Carbidopa makes more levodopa available to the brain. 4. Carbidopa crosses the blood-brain barrier to treat Parkinson's disease.

3. Carbidopa enhances the effects of levodopa by inhibiting decarboxylase in the periphery, thereby making more levodopa available to the central nervous system. Sinemet is the most effective treatment for PD

Which is a common cognitive problem associated with Parkinson's disease? 1. Emotional lability. 2. Depression. 3. Memory deficits. 4. Paranoia.

3. Memory deficits are cognitive impairments. The client may also develop a dementia.

The client diagnosed with an acute exacerbation of multiple sclerosis is placed on high-dose intravenous injections of corticosteroid medication. Which nursing intervention should be implemented? 1. Discuss discontinuing the proton pump inhibitor with the HCP. 2. Hold the medication until after all cultures have been obtained. 3. Monitor the client's serum blood glucose levels frequently. .4. Provide supplemental dietary sodium with the client's meals.

3. Monitor the client's serum blood glucose levels frequently. Steroids interfere with glucose metabolism by blocking the action of insulin;therefore, the blood glucose levels should be monitored. 1. Steroid medications increase gastric acid;therefore, a proton pump inhibitor is an appropriate medication for the client. 2. Cultures are ordered prior to administer-ing antibiotics, not steroids. 4. Steroid medications cause the client to retain sodium; therefore, a low-sodiumdiet should be encouraged

The client diagnosed with hypothyroidism is prescribed the thyroid hormone levothyroxine (Synthroid). Which assessment data indicate the medication has been effective? 1. The client has a three (3)-pound weight gain. 2. The client has a decreased pulse rate. 3. The client's temperature is WNL. 4. The client denies any diaphoresis.

3.The client with hypothyroidism frequently has a subnormal temperature,so a temperature WNL indicates the medication is effective. 1. The medication will help increase the client's metabolic rate. A weight gain indicates not enough medication is being taken to put the client in a euthyroid (normal thyroid) state. 2. A decreased pulse rate indicates there is not enough thyroid hormone level; therefore, the medication is not effective. 4. Diaphoresis (sweating) occurs with hyper-thyroidism, not hypothyroidism

A family member asks the nurse to explain the purpose of hospice care. Which of the following is the best response? Hospice care: 1) Is appropriate when the patient desires to intentionally end his life 2) Focuses on minimizing the disease process as rapidly as possible 3) Focuses on symptom management for patients not responding to treatment 4) Is holistic care for patients dying or debilitated and not expected to improve

4) Is holistic care for patients dying or debilitated and not expected to improve Rationale: Hospice care focuses on holistic care of patients actively dying or not expected to improve. It helps patients face death with dignity and comfort. Euthanasia refers to the deliberate ending of a life. Palliative care is aggressively planned care that manages symptoms of patients whose disease process no longer responds to treatment. Aggressive medical treatment is aimed at stopping the disease process.

The client is scheduled to have a kidney, ureter, and bladder (KUB) radiograph. To prepare the client for this procedure, the nurse should explain to the client that: 1. Fluid and food will be withheld the morning of the examination. 2. A tranquilizer will be given before the examination. 3. An enema will be given before the examination. 4. No special preparation is required for the examination.

4. A KUB radiographic examination ordinarily requires no preparation. It is usually done while the client lies supine and does not involve the use of radiopaque substances.

When providing family education for those who have a relative with Alzheimer's disease about minimizing stress, which of the following suggestions is most relevant? 1. Allow the client to go to bed four to five times during the day. 2. Test the cognitive functioning of the client several times a day. 3. Provide reality orientation even if the memory loss is severe. 4. Maintain consistency in environment, routine, and caregivers.

4. Change increases stress. Therefore, the most important and relevant suggestion is to maintain consistency in the client's environment, routine, and caregivers. Although rest periods are important, going to bed interferes with the sleep-wake cycle. Rest in a recliner chair is more useful. Testing cognitive functioning and reality orientation are not likely to be successful and may increase stress if memory loss is severe.

The nurse caring for a client diagnosed with Parkinson's disease writes a problem of "impaired nutrition." Which nursing intervention would be included in the plan of care? 1. Consult the occupational therapist for adaptive appliances for eating. 2. Request a low-fat, low-sodium diet from the dietary department. 3. Provide three (3) meals per day that include nuts and whole-grain breads. 4. Offer six (6) meals per day with a soft consistency.

4. The client's energy levels will not sustain eating for long periods. Offering frequent and easy-to-chew (soft) meals of small proportions is the preferred dietary plan.

A client has a urinary catheter and continuous bladder irrigation after a transurethral resection of the prostate this morning. The amount of bladder irrigating solution that has infused over the past 12 hours is 1000 mL. The amount of fluid in the urinary drainage bag is 1725 mL. The nurse records that the client had ____ mL urinary output in the past 12 hours.

725

The order for a patient who has a severe case of shingles is for acyclovir (Zovirax) 10 mg/kg IV every 8 hours for 7 days. The patient weighs 165 pounds. How much is each dose?

750 mg per dose

While conversing with a patient who had a stroke six months ago, you note their speech is hard to understand and slurred. This is known as: A. Dysarthria B. Apraxia C. Alexia D. Dysphagia

A

A client who was tested for human immunodeficiency virus (HIV) after a recent exposure had a negative result. During the post-test counseling session, the nurse tells the client which of the following? a) the test should be repeated in 6 months b) this ensures that the client is not infected with the HIV virus c) the client no longer needs to protect himself from sexual partners d) the client probably has immunity to the acquired immunodeficiency virus

A A negative test result indicates that no HIV antibodies were detected in the blood sample. A repeated test in 6 months is recommended because false-negative test results have occurred early in the infection. Options B, C, and D are incorrect.

During the past 6 months, a client diagnosed with acquired immunodeficiency syndrome has had chronic diarrhea and has lost 18 pounds. Additional assessment findings include tented skin turgor, dry mucous membranes, and listleness. Which nursing diagnosis focuses attention on the client's most immediate problem? A. Deficient fluid volume related to diarrhea and abnormal fluid loss B. Imbalanced nutrition: less than body requirements related to nausea and vomiting C. Disturbed thought processes related to central nervous system effects of disease D. Diarrhea related to the disease process and acute infection

A Based on the client's assessment findings, the most immediate problem is dehydration because of chronic diarrhea. The nursing diagnosis of deficient fluid volume is the priority, and interventions are geared to improving the client's fluid status. Although imbalanced nutrition, disturbed thought processes, and diarrhea are involved, they assume a lower priority at this time.

A client was admitted with iron deficiency anemia and blood-streaked emesis. Which question is most appropriate for the nurse to ask in determining the extent of the client's activity intolerance? A. "What activities were you able to do 6 months ago compared with the present?" B. "How long have you had this problem?" C. "Have you been able to keep up with all your usual activities?" D. "Are you more tired now than you used to be?"

A It is difficult to determine activity intolerance without objectively comparing activities from one time frame to another. Because iron deficiency anemia can occur gradually and individual endurance varies, the nurse can best assess the client's activity tolerance by asking the client to compare activities 6 months ago and at the present.

Which clients are at risk for acute kidney injury (AKI)? (Select all that apply.) A Football player in preseason practice Correct B Client who underwent contrast dye radiology Correct C Accident victim recovering from a severe hemorrhage Correct D Accountant with diabetes E Client in the intensive care unit on high doses of antibiotics Correct F Client recovering from gastrointestinal influenza

A Football player in preseason practice B Client who underwent contrast dye radiology C Accident victim recovering from a severe hemorrhage E Client in the intensive care unit on high doses of antibiotics F Client recovering from gastrointestinal influenza To prevent AKI, all people should be urged to avoid dehydration by drinking at least 2 to 3 liters of fluids daily, especially during strenuous exercise or work associated with diaphoresis, or when recovering from an illness that reduces kidney blood flow, such as influenza. Contrast media may cause acute renal failure, especially in older clients with reduced kidney function. Recent surgery or trauma, transfusions, or other factors that might lead to reduced kidney blood flow may cause AKI. Certain antibiotics may cause nephrotoxicity. Diabetes may cause acute kidney failure superimposed on chronic kidney failure.

Which clinical manifestation indicates the need for increased fluids in a client with kidney failure? A Increased blood urea nitrogen (BUN) B Increased creatinine level C Pale-colored urine D Decreased sodium level

A Increased blood urea nitrogen (BUN An increase in BUN can be an indication of dehydration, and an increase in fluids is needed. Increased creatinine indicates kidney impairment. Urine that is pale in color is diluted; an increase in fluids is not necessary. Sodium is increased, not decreased, with dehydration.

The nurse teaches a client who is recovering from acute kidney disease to avoid which type of medication? A Nonsteroidal anti-inflammatory drugs (NSAIDs) B Angiotensin-converting enzyme (ACE) inhibitors C Opiates D Calcium channel blockers filtration rate and blood flow within the kidney.

A Nonsteroidal anti-inflammatory drugs (NSAIDs) NSAIDs may be nephrotoxic to a client with acute kidney disease, and should be avoided. ACE inhibitors are used for treatment of hypertension and to protect the kidneys, especially in the diabetic client, from progression of kidney disease. Opiates may be used by clients with kidney disease if severe pain is present; however, excretion may be delayed. Calcium channel blockers can improve the glomerular

While managing care for a client with chronic kidney disease, which actions does the registered nurse (RN) plan to delegate to unlicensed assistive personnel (UAP)? (Select all that apply.) A Obtain the client's pre-hemodialysis weight. ] B Check the arteriovenous (AV) fistula for a thrill and bruit. C Document the amount the client drinks throughout the shift. D Auscultate the client's lung sounds every 4 hours. E Explain the components of a low-sodium diet.

A Obtain the client's pre-hemodialysis weight. C Document the amount the client drinks throughout the shift. Obtaining the client's weight and documenting oral fluid intake are routine tasks that can be performed by UAP. Assessment skills (checking the AV fistula and auscultating lung sounds) and client education (explaining special diet) require more education and are in the legal scope of practice of the RN.

The nurse assists a client with acute kidney injury (AKI) to modify the diet in which ways? (Select all that apply.) A Restricted protein B Liberal sodium C Restricted fluids D Low potassium E Low fat

A Restricted protein C Restricted fluids D Low potassium Breakdown of protein leads to azotemia and increased blood urea nitrogen. Fluid is restricted during the oliguric stage. Potassium intoxication may occur, so dietary potassium is also restricted. Sodium is restricted during AKI because oliguria causes fluid retention. Fats may be used for needed calories when proteins are restricted.

A patient who is near death has the following nursing diagnosis: Impaired gas exchange related to fluid in the lungs. Which of these interventions is most appropriate? A ~ Maintain patient in a side-lying position. B ~ Suction the patient as needed. C ~ Teach the patient how to do pursed-lip breathing. D ~ Encourage the patient to cough and deep-breathe at least every 3 hours.

A ~ A side-lying position will help prevent aspiration of mucus and fluids produced during the dying process. Suctioning is uncomfortable and will stimulate more mucus and fluid production by irritating the mucosa. Pursed-lip breathing and coughing will not decrease the mucus and fluid production and would be exhausting to the dying patient.

The nurse teaches the client that the major difference between angina and pain associated with myocardial infarction (MI) is that: A) Angina is relieved with nitroglycerin and rest. B) Angina can be fatal. C) MI pain always radiates to the left arm or jaw. D) MI pain cannot be treated.

A) Angina is relieved with nitroglycerin and rest.

Which of the following classes of medications protects the ischemic myocardium by blocking catecholamines and sympathetic nerve stimulation? A) Beta-adrenergic blockers B) Calcium channel blockers C) Narcotics D) Nitrates

A) Beta-adrenergic blockers Beta-adrenergic blockers work by blocking beta receptors in the myocardium, reducing the response to catecholamines and sympathetic nerve stimulation. They protect the myocardium, helping to reduce the risk of another infarction by decreasing myocardial oxygen demand. Calcium channel blockers reduce the workload of the heart by decreasing the heart rate. Narcotics reduce myocardial oxygen demand, promote vasodilation, and decrease anxiety. Nitrates reduce myocardial oxygen consumption by decreasing left ventricular end-diastolic pressure (preload) and systemic vascular resistance (afterload).

The patient with cirrhosis has an increased abdominal girth from ascites. The nurse should know that this fluid gathers in the abdomen for which reasons (select all that apply)? A. There is decreased colloid oncotic pressure from the liver's inability to synthesize albumin. B. Hyperaldosteronism related to damaged hepatocytes increases sodium and fluid retention. C. Portal hypertension pushes proteins from the blood vessels, causing leaking into the peritoneal cavity. D. Osmoreceptors in the hypothalamus stimulate thirst, which causes the stimulation to take in fluids orally. E. Overactivity of the enlarged spleen results in increased removal of blood cells from the circulation, which decreases the vascular pressure.

A, B, C A. There is decreased colloid oncotic pressure from the liver's inability to synthesize albumin. B. Hyperaldosteronism related to damaged hepatocytes increases sodium and fluid retention. C. Portal hypertension pushes proteins from the blood vessels, causing leaking into the peritoneal cavity. Correct The ascites related to cirrhosis are caused by decreased colloid oncotic pressure from the lack of albumin from liver inability to synthesize it and the portal hypertension that shifts the protein from the blood vessels to the peritoneal cavity, and hyperaldosteronism which increases sodium and fluid retention. The intake of fluids orally and the removal of blood cells by the spleen do not directly contribute to ascites.

The nurse working with oncology clients understands that interacting factors affect cancer development. Which factors does this include? (Select all that apply.) a. Exposure to carcinogens b. Genetic predisposition c. Immune function d. Normal doubling time e. State of euploidy

A, B, C ~ The three interacting factors needed for cancer development are exposure to carcinogens, genetic predisposition, and immune function.

The nurse is reviewing the history of a patient suspected of having hyperthyroidism. Which manifestation(s) would be supportive of the diagnosis? (Select all that apply.) A. Increased heart rate B. Increased appetite C. Emotional instability D. Mental sluggishness E. Hyperactivity with increasing sense of fatigue

A, B, C, E The earliest symptoms of hyperthyroidism may be weight loss (in spite of a good appetite) and nervousness. Symptoms can vary from mild to severe, and may include weakness, insomnia, tremulousness, agitation, tachycardia, palpitations, exertion-related dyspnea, ankle edema, difficulty concentrating, diarrhea, increased thirst and urination, decreased libido, scanty menstruation, and infertility. The condition is sometimes not diagnosed in its early stages because of the vagueness of the symptoms. In some cases, hyperthyroidism is misdiagnosed as a cardiovascular disease because symptoms for both conditions are similar. Hypothyroidism is associated with mental sluggishness.

When caring for a patient with liver disease, the nurse recognizes the need to prevent bleeding resulting from altered clotting factors and rupture of varices. Which nursing interventions would be appropriate to achieve this outcome (select all that apply)? A. Use smallest gauge needle possible when giving injections or drawing blood. B. Teach patient to avoid straining at stool, vigorous blowing of nose, and coughing. C. Advise patient to use soft-bristle toothbrush and avoid ingestion of irritating food. D. Apply gentle pressure for the shortest possible time period after performing venipuncture. E. Instruct patient to avoid aspirin and NSAIDs to prevent hemorrhage when varices are present.

A, B, C, E A. Use smallest gauge needle possible when giving injections or drawing blood. B. Teach patient to avoid straining at stool, vigorous blowing of nose, and coughing. C. Advise patient to use soft-bristle toothbrush and avoid ingestion of irritating food. E. Instruct patient to avoid aspirin and NSAIDs to prevent hemorrhage when varices are present. Using the smallest gauge needle for injections will minimize the risk of bleeding into the tissues. Avoiding straining, nose blowing, and coughing will reduce the risk of hemorrhage at these sites. The use of a soft-bristle toothbrush and avoidance of irritating food will reduce injury to highly vascular mucous membranes. The nurse should apply gentle but prolonged pressure to venipuncture sites to minimize the risk of bleeding. Aspirin and NSAIDs should not be used in patients with liver disease because they interfere with platelet aggregation, thus increasing the risk for bleeding.

A nurse is providing community education on the seven warning signs of cancer. Which signs are included? (Select all that apply.) a. A sore that does not heal b. Changes in menstrual patterns c. Indigestion or trouble swallowing d. Near-daily abdominal pain e. Obvious change in a mole

A, B, C, E ~ The seven warning signs for cancer can be remembered with the acronym CAUTION: changes in bowel or bladder habits, a sore that does not heal, unusual bleeding or discharge, thickening or lump in the breast or elsewhere, indigestion or difficulty swallowing, obvious change in a wart or mole, and nagging cough or hoarseness. Abdominal pain is not a warning sign.

Which of the following symptoms you as the nurse expect to see in the patient with primary progressive multiple sclerosis? (Select All that Apply): A) Unilateral Vision Loss B) Fatigue C) Diarrhea D) Intention tremors E) Paralytic ileus

A, B, D

Which are risk factors that are known to contribute to atherosclerosis-related diseases? (Select all that apply.) A. Low-density lipoprotein cholesterol (LDL-C) of 160 mg/dL B. Smoking C. Aspirin (acetylsalicylic acid [ASA]) consumption D. Type 2 diabetes E. Vegetarian diet

A, B, D Having an LDL-C value of less than 100 mg/dL is optimal; 100 to 129 mg/dL is near or less than optimal; with LDL-C 130 to 159 mg/dL (borderline high), the client is advised to modify diet and exercise. Smoking is a modifiable risk factor and should be avoided or terminated, and diabetes is a risk factor for atherosclerotic disease.

A patient with a 10-year history of angina is admitted to the unit with chest pain. The working medical diagnosis is to rule out MI. Definitive diagnosis of MI will be based on which information? (Select all that apply.) A. ECG B. Cardiac enzyme levels C. Fluid and electrolyte status D. Patient history and physical exam E. CBC (complete blood count) with differential and ABGs (arterial blood gases)

A, B, D Patient history is suggestive of MI; cardiac enzyme levels will be the most significant data, along with troponin levels, for diagnosing MI; ECG will show areas of damage as a result of the infarct. Fluid and electrolyte status as well as CBC with differential and ABGs may be obtained to gather more information about the patient's clinical status, but these are not used to diagnose an MI.

You're working on a medical surgical floor. You have the following patients below. Select all the patients below that are at risk for a pressure injury: A. A 19 year old female who is a quadriplegic. B. A 35 year old male with a BMI of 13.6 that is incontinent of stool and has a right leg splint. C. A 55 year old female who has controlled diabetes and is ambulating three times a day. D. A 76 year old male with an elevated ammonia level and is excessively sweaty. E. A 45 year old with a Braden Scale score of 7.

A, B, D, and E. The only patient not at risk for a pressure injury is the patient in option C. Remember altered sensory perception, any type of moisture issue (incontinence, sweating etc.), immobility, poor nutrition, altered mental status (high ammonia level can cause confusion and drowsiness), Braden scale score less than 9 are all risk factors for a pressure injury.

A patient is scheduled to receive external radiation therapy. Which potential side effects should the nurse include in patient teaching points? Select all that apply. A ~ Skin reaction at site of exposure B ~ Loss of appetite C ~ Mental slowness D ~ Fatigue E ~ Potential contamination to others

A, B, D, ~ External radiation therapy has far fewer side effects than in the past but does include skin reaction, loss of appetite, and fatigue. Mental slowness is not a side effect of external radiation therapy. Contamination to others can occur with internal radiation therapy.

A patient is bought in by ambulance with a suspected brain injury. What are the outward symptoms of head injury? (Select all that apply.) A. Tinnitus B. Diarrhea C. Ottorhea D. Battle sign

A, C, D The outward symptoms of head injury include tinnitus, ottorhea, and Battle sign. Diarrhea is not a symptom of head injury.

A 57-year-old man is admitted with a diagnosis of cirrhosis. The nurse is aware that he will most likely require which intervention(s)? (Select all that apply.) A. Diuretics B. Increased fluids C. Bleeding precautions D. Vegetable-based proteins E. Lactulose administration

A, C, D, E Because the liver produces clotting factors and is now dysfunctional, risk for bleeding exists. The liver cannot metabolize proteins, especially albumin, properly. This leads to edema and ascites and requires diuretics, preferably potassium wasting. Ammonia buildup is likely; lactulose binds with this toxic metabolic by-product and allows for its excretion through the GI tract. Patients with liver disorders are at high risk for fluid volume excess.

You are performing discharge teaching with a patient who is going home on Synthroid. Which statement by the patient causes you to re-educate the patient about this medication? A. "I will take this medication at bedtime with a snack." B. "I will never stop taking the medication abruptly." C. "If I have palpitations, chest pain, intolerance to heat, or feel restless, I will notify the doctor." D. "I will not take this medication at the same time I take my Carafate."

A. "I will take this medication at bedtime with a snack." Synthroid is best taken in the MORNING on an empty stomach. All the other statements are correct about taking Synthroid.

Which statement regarding continuous ambulatory peritoneal dialysis (CAPD) is of highest priority when teaching a patient new to this procedure? A. "It is essential that you maintain aseptic technique to prevent peritonitis." B. "You will be allowed a more liberal protein diet after you complete CAPD." C. "It is important for you to maintain a daily written record of blood pressure and weight." D. "You must continue regular medical and nursing follow-up visits while performing CAPD."

A. "It is essential that you maintain aseptic technique to prevent peritonitis." Peritonitis is a potentially fatal complication of peritoneal dialysis, and it is imperative to teach the patient methods to prevent it from occurring. Although the other teaching statements are accurate, they do not address the potential for mortality by peritonitis, making that nursing action the highest priority.

The student nurse is reviewing the patient's chart and notes that the patient experiences intermittent claudication. When is the patient most likely to experience this disorder? A. After exercise B. Immediately after exposure to cold C. First thing in the morning after getting out of bed D. After dorsiflexion of the foot when phlebitis is present

A. After exercise Intermittent claudication is inadequate arterial supply to muscles in the lower extremities that results in severe cramping of the muscles after activity. Dorsiflexion of the foot when phlebitis is present is a positive Homan sign, indicating the possible presence of deep vein thrombosis (DVT). However, it is no longer considered a reliable assessment for DVT.

The client has a traumatic brain injury from a motor vehicle accident. Which sign does the nurse associate with increased intracranial pressure (ICP)? A. Changes in breathing pattern B. Dizziness when sitting up C. Increasing level of consciousness D. Equal and reactive pupils

A. Changes in breathing pattern

The HIV-infected patient is taught health promotion activities including good nutrition; avoiding alcohol, tobacco, drug use, and exposure to infectious agents; keeping up to date with vaccines; getting adequate rest; and stress management. What is the rationale behind these interventions that the nurse knows? A. Delaying disease progression B. Preventing disease transmission C. Helping to cure the HIV infection D. Enabling an increase in self-care activities

A. Delaying disease progression These health promotion activities along with mental health counseling, support groups, and a therapeutic relationship with health care providers will promote a healthy immune system, which may delay disease progression. These measures will not cure HIV infection, prevent disease transmission, or increase self-care activities

The nurse is caring for a patient who was just admitted to the hospital with an acute MI. What complication is the nurse most concerned will occur with this patient? A. Dysrhythmia B. Hyperkalemia C. Respiratory failure D. Hypovolemic shock

A. Dysrhythmia The most common complication following an MI is dysrhythmia due to increased myocardial irritability, which can be fatal. The patient would develop cardiogenic, not hypovolemic, shock. Hyperkalemia and respiratory failure are not the nurse's greatest concerns at this time.

After a head injury, the patient begins to have drainage from the nose. The nurse assesses the drainage by which method? A. Halo test B. Tinel sign C. Battle sign D. Babinski sign

A. Halo test The halo test is used to determine whether drainage from the nose or ear is cerebrospinal fluid. Tinel sign is one assessment used during the assessment of carpal tunnel symptoms. Bruising behind the ear that occurs after a head injury is called Battle sign. Babinski sign is checked as part of a neurologic assessment.

The GCS score for your client with a risk for increased intracranial pressure has been stable at 12 for the last 6 hours. This time you rate him at 9. Which of the following have you noted and what does it mean? A. He is less responsive, a sign that his intracranial pressure may be increasing B. He is more responsive, a sign that he may be improving C. His pupils are fixed and dilated, an ominous sign D. He does not move or make sounds, which may mean he got too much pain medication

A. He is less responsive, a sign that his intracranial pressure may be increasing

A patient with COPD asks the nurse to turn his oxygen up from 3 L/min via nasal cannula to 5 L/min. The nurse explains to the patient that she cannot turn his oxygen up this high. What is the reason the oxygen cannot be increased to 5 L/min? A. Hypoxic drive is necessary for breathing. B. Hypercapnic drive is necessary for breathing. C. Higher concentrations may result in a severe headache. D. Higher levels will be required later for arterial blood gases (ABGs).

A. Hypoxic drive is necessary for breathing. Respiratory effort is stimulated by hypoxemia in patients with COPD. Higher oxygen levels would eliminate the stimulus to breathe. Low oxygen levels, not high carbon dioxide levels, are the stimulus for breathing for a patient with COPD. Higher concentrations of oxygen will not be required for ABGs or result in a headache.

The patient presents to the clinic with a gross enlargement of the anterior neck. The nurse knows that this is most likely caused by a deficiency in which substance? A. Iodine B. Sodium C. Calcium D. Vitamin D

A. Iodine A gross enlargement of the thyroid gland in the anterior neck is likely a goiter caused by iodine deficiency. The patient's symptoms are not caused by sodium, calcium, or vitamin D deficiencies.

A patient who is admitted to the coronary care unit with an acute MI is complaining of severe substernal chest pain. What is the cause of the chest pain? A. Ischemia of the myocardium B. Ischemia of the carotid artery C. Spasm of the coronary arteries D. Vasodilation of the coronary veins

A. Ischemia of the myocardium Myocardial ischemia causes crushing substernal pain, typically radiating to the left arm and/or left side of the jaw. Coronary artery spasm produces anginal pain. If it continues, however, it can lead to an MI. Crushing substernal pain is not caused by ischemia of the carotid artery or vasodilation of the coronary veins.

Which statement(s) is/are most accurate regarding transmission of human immunodeficiency virus (HIV) and development of acquired immune deficiency syndrome (AIDS)? (Select all that apply.) A. It is treatable. B. Poor personal hygiene is a factor in its transmission. C. It cannot be transmitted if safer sexual practices are used. D. It enters the bloodstream through breaks in mucous membrane. E. It is a blood-borne pathogen and can be transmitted by contaminated intravenous (IV) drug use supplies.

A. It is treatable. D. It enters the bloodstream through breaks in mucous membrane. E. It is a blood-borne pathogen and can be transmitted by contaminated intravenous (IV) drug use supplies. HIV/AIDS is a treatable viral infection that enters the body through breaks in mucous membranes (e.g., oral, vaginal, rectal mucosa). Although HIV is a blood-borne pathogen, use of contaminated IV drug use supplies is one of many ways it is transmitted. IV drug use significantly increases the risk of HIV infection. Although risk of infection is considerably lessened by use of safer sex practices, intimate sexual contact involving any mucous membrane will pose a risk of transmission. Personal hygiene has no effect on susceptibility to or transmission of HIV.

The nurse is caring for a patient with late-stage cirrhosis. The nurse considers which factor when participating in a patient care conference? A. Late-stage cirrhosis is irreversible. B. Late-stage cirrhosis can be managed with lifestyle changes. C. Late-stage cirrhosis can be cured with lactulose and spironolactone. D. Late-stage cirrhosis is characterized by periods of remission alternating with flare-ups.

A. Late-stage cirrhosis is irreversible. After cirrhosis reaches the late stage, it is irreversible. Lactulose is used to manage ammonia levels and hepatic encephalopathy, but it is not curative. Cirrhosis does not have remission periods.

Nurses need to educate patients at risk for CKD. Which individuals are considered to be at increased risk (select all that apply)? A. Older African Americans B. Individuals older than 60 years C. Those with a history of pancreatitis D. Those with a history of hypertension E. Those with a history of type 2 diabetes

A. Older African Americans B. Individuals older than 60 years D. Those with a history of hypertension E. Those with a history of type 2 diabetes Risk factors for CKD include diabetes mellitus, hypertension, age older than 60 years, cardiovascular disease, family history of CKD, exposure to nephrotoxic drugs, and ethnic minorities (e.g., African American, Native American).

When planning care for the patient with acute pancreatitis, the LPN/LVN knows that which intervention is a priority of care? A. Pain control B. Nutritional supplementation C. Observation for mental changes D. Observation for intestinal obstruction

A. Pain control The patient with acute pancreatitis presents with pain. The intervention having the highest priority involves management of the pain. Nutritional supplementation and observation for mental changes and intestinal obstruction are appropriate interventions, but not the ones of highest importance.

A patient is admitted to the hospital with CKD. You understand that this condition is characterized by A. Progressive irreversible destruction of the kidneys B. A rapid decrease in urinary output with an elevated BUN level C. Increasing creatinine clearance with a decrease in urinary output D. Prostration, somnolence, and confusion with coma and imminent death

A. Progressive irreversible destruction of the kidneys CKD involves progressive, irreversible loss of kidney function.

A patient who is in her first trimester of pregnancy is diagnosed with hyperthyroidism. Which medication do you suspect the patient will be started on? A. Propylthiouracil (PTU) B. Radioactive Iodine C. Tapazole D. Synthroid

A. Propylthiouracil (PTU) Propylthiouracil (PTU) is the only anti-thyroid medication that can be used during the 1st trimester of pregnancy.

The nurse is caring for a client with peripheral arterial disease (PAD). For which symptoms does the nurse assess? A. Reproducible leg pain with exercise B. Unilateral swelling of affected leg C. Decreased pain when legs are elevated D. Pulse oximetry reading of 90%

A. Reproducible leg pain with exercise Claudication (leg pain with ambulation due to ischemia) is reproducible in similar circumstances. Unilateral swelling is typical of venous problems such as deep vein thrombosis. With PAD, pain decreases with legs in the dependent position. Pulse oximetry readings reflect the amount of oxygen bound to hemoglobin; PAD results from atherosclerotic occlusion of peripheral arteries.

Which patient is the nurse most concerned with developing a subdural hematoma following an injury that resulted with a blow to the head? A. The 76-year-old patient who is taking an anticoagulant B. The 16-year-old football player who suffered a concussion C. The 36-year-old patient who has a history of migraine headaches D. The 56-year-old patient who is taking an antihypertensive medication

A. The 76-year-old patient who is taking an anticoagulant A subdural hematoma results when blood leaks under the dura mater (subdural) and presses against the softer arachnoid membrane and the brain tissue it is covering. As blood leaks, the hematoma grows in size. The 76-year-old patient is most at risk for a subdural hematoma due to his anticoagulant medication. The football player who suffered a concussion is at an increased risk for a head bleed, but less so than the elderly patient taking anticoagulant medication. The patients with migraine headaches and antihypertensive medications are not at an increased risk for hemorrhage.

A patient who has tested positive for the human immunodeficiency virus (HIV) arrives at the clinic with a report of fever, nonproductive cough, and fatigue. The patient's CD4 count is 184 cells/mcL. How should the healthcare provider interpret these findings? Please choose from one of the following options. A. The patient is diagnosed with acquired immunodeficiency syndrome (AIDS). B.The patient is now in the latent stages of HIV infection C.These findings provide evidence that the patient has seroconverted. D. This is an expected finding because the patient has tested positive for HIV.

A. The patient is diagnosed with acquired immunodeficiency syndrome (AIDS).

A 54-year-old man is admitted to the cardiac unit with chest pain radiating to his jaw and left arm. Which enzyme would be most specific in the diagnosis of a myocardial infarction (MI)? A. Troponin B. CK-MB (creatine kinase MB) C. LDH (L-lactate dehydrogenase) D. AST (aspartate aminotransferase)

A. Troponin The most significant laboratory finding for diagnosis of MI is an elevated troponin level, especially if accompanied by an elevated CK-MB. Troponin is found only in cardiac tissue. LDH and AST are not the most significant laboratory findings for the diagnosis of MI.

How is the effectiveness of antiviral drugs administered to treat HIV infection assessed and evaluated? A. Viral load B. Megakaryocytes C. Lymphocyte counts D. Red blood cell counts

A. Viral load All antiretroviral drugs work to reduce the viral load, which is the number of viral RNA copies per milliliter of blood.

A female patient with systemic lupus erythematosus (SLE) complains of photosensitivity. The LPN/LVN should instruct the patient on which self-care measure? A. Wear sunblock whenever she is outdoors. B. Include foods high in beta-carotene in her diet. C. Manage joint pain with prescribed medications. D. Use a tanning bed once per week to help with rashes.

A. Wear sunblock whenever she is outdoors. The patient with SLE should use a sunblock with an SPF of 30 or higher because direct sunlight can cause skin rashes and a generalized flare-up of the condition. Tanning beds should always be avoided. Foods high in beta-carotene will not help photosensitivity. Joint pain should be managed, but these medications will not prevent photosensitivity.

When educating a patient about wound healing the nurse should include what in the teaching? A. inadequate nutrition delays wound healing and increases risk of infection. B. chronic wounds heal better in a dry, open environment so leave them open to air. C. fat tissue heals more rapidly because there is less vascularization. D. long term steroid use diminishes the inflammatory response and speeds up wound healing

A. inadequate nutrition delays wound healing and increases risk of infection

What evaluation criteria are included in the Braden Risk assessment? (select all that apply) A. sensory perception B. medications C. mobility D. friction and shear E. mental status F. moisture

A. sensory perception C. mobility D. friction and shear F. moisture

What strategies should be included in pressure ulcer prevention (select all that apply) A. use moisture barrier ointment with incontinence B. reposition immobile patients every 4 hours C. when patient in side lying position ensure HOB <30 degrees D. place patient on pressure reducing support surface E. maintain bed at 45 degree angle F. massage reddened bony prominences G. oral nutrition supplement should be used when undernourished.

A. use moisture barrier ointment with incontinence C. when patient in side lying position ensure HOB <30 degrees D. place patient on pressure reducing support surface G. oral nutrition supplement should be used when undernourished.

A patient with amyotrophic lateral sclerosis (ALS) is hospitalized with pneumonia. Which nursing action will be included in the plan of care? a. Assist with active range of motion. b. Observe for agitation and paranoia. c. Give muscle relaxants as needed to reduce spasms. d. Use simple words and phrases to explain procedures.

ANS: A ALS causes progressive muscle weakness, but assisting the patient to perform active ROM will help to maintain strength as long as possible. Psychotic symptoms such as agitation and paranoia are not associated with ALS. Cognitive function is not affected by ALS, and the patient's ability to understand procedures will not be impaired. Muscle relaxants will further increase muscle weakness and depress respirations.

The nurse at the clinic is interviewing a 64-year-old woman who is 5 feet, 3 inches tall and weighs 125 pounds (57 kg). The patient has not seen a health care provider for 20 years. She walks 5 miles most days and has a glass of wine 2 or 3 times a week. Which topics will the nurse plan to include in patient teaching about cancer screening and decreasing cancer risk (select all that apply)? a. Pap testing b. Tobacco use c. Sunscreen use d. Mammography e. Colorectal screening

ANS: A, C, D, E The patient's age, gender, and history indicate a need for screening and/or teaching about colorectal cancer, mammography, Pap smears, and sunscreen. The patient does not use excessive alcohol or tobacco, she is physically active, and her body weight is healthy.

A patient is seen in the health clinic with symptoms of a stooped posture, shuffling gait, and pill rolling-type tremor. The nurse will anticipate teaching the patient about a. oral corticosteroids. b. antiparkinsonian drugs. c. the purpose of electroencephalogram (EEG) testing. d. preparation for magnetic resonance imaging (MRI).

ANS: B The diagnosis of Parkinson's is made when two of the three characteristic signs of tremor, rigidity, and bradykinesia are present. The confirmation of the diagnosis is made on the basis of improvement when antiparkinsonian drugs are administered. This patient has symptoms of tremor and bradykinesia; the next anticipated step will be treatment with medications. MRI and EEG are not useful in diagnosing Parkinson's disease, and corticosteroid therapy is not used to treat it.

A patient who is scheduled for a right breast biopsy asks the nurse the difference between a benign tumor and a malignant tumor. Which answer by the nurse is correct? a. "Benign tumors do not cause damage to other tissues." b. "Benign tumors are likely to recur in the same location." c. "Malignant tumors may spread to other tissues or organs." d. "Malignant cells reproduce more rapidly than normal cells."

ANS: C The major difference between benign and malignant tumors is that malignant tumors invade adjacent tissues and spread to distant tissues and benign tumors never metastasize. The other statements are inaccurate. Both types of tumors may cause damage to adjacent tissues. Malignant cells do not reproduce more rapidly than normal cells. Benign tumors do not usually recur.

Which information about a patient who is being treated with carbidopa/levodopa (Sinemet) for Parkinson's disease is most important for the nurse to report to the health care provider? a. Shuffling gait b. Tremor at rest c. Cogwheel rigidity of limbs d. Uncontrolled head movement

ANS: D Dyskinesia is an adverse effect of the Sinemet, indicating a need for a change in medication or decrease in dose. The other findings are typical with Parkinson's disease

To determine the severity of the symptoms for a patient with benign prostatic hyperplasia (BPH), the nurse will ask the patient about a. blood in the urine. b. lower back or hip pain. c. erectile dysfunction (ED). d. strength of the urinary stream.

ANS: D The American Urological Association (AUA) Symptom Index for a patient with BPH asks questions about the force and frequency of urination, nocturia, etc. Blood in the urine, ED, and back or hip pain are not typical symptoms of BPH.

A patient with Parkinson's disease has decreased tongue mobility and an inability to move the facial muscles. Which nursing diagnosis is of highest priority? a. Activity intolerance b. Self-care deficit: toileting c. Ineffective self-health management d. Imbalanced nutrition: less than body requirements

ANS: D The data about the patient indicate that poor nutrition will be a concern because of decreased swallowing. The other diagnoses also may be appropriate for a patient with Parkinson's disease, but the data do not indicate they are current problems for this patient.

The nurse reviews the laboratory results of a patient who is receiving chemotherapy. Which laboratory result is most important to report to the health care provider? a. Hematocrit of 30% b. Platelets of 95,000/µL c. Hemoglobin of 10 g/L d. White blood cell (WBC) count of 2700/µL

ANS: D The low WBC count places the patient at risk for severe infection and is an indication that the chemotherapy dose may need to be lower or that WBC growth factors such as filgrastim (Neupogen) are needed. Although the other laboratory data indicate decreased levels, they do not indicate any immediate life-threatening adverse effects of the chemotherapy.

The LPN is providing discharge information to a client with hep B. The LPN instructs the client to prevent transmission via: a. airborne pathogens 2. blood and body secretions 3. skin contact 4. fecal and oral routes

Answer 2 Hep b is transmitted via blood and body secretions. The LPN instructs the client to prevent transmission through correct use of latex condoms, and by not sharing personal care items that may have blood on them. Diseases such as pneumonia are spread by airborne pathogens, hep A is spread by fecal and oral routes. Hep B is not transmitted by skin contact.

The nurse sticks herself with a dirty needle. Which action should the nurse implement first? 1.Notify the infection control nurse. 2.Cleanse the area with soap and water. 3.Request post-exposure prophylaxis. 4.Check the hepatitis status of the client.

Answer 2. The nurse should first clean the needle stick with soap and water to help remove any virus that is on the skin

Which statement by the client diagnosed with hepatitis warrants immediate intervention by the clinic nurse? 1) I will not drink any type of beer or mixed drink. 2)I will get adequate rest so I don't get exhausted. 3) I had a big hearty breakfast this morning. 4) I took some cough syrup for this nasty head cold.

Answer 4: Rationale: 1) The client should avoid all alcohol to prevent further liver damage and promote healing. 2) Rest is needed for healing of the liver and to promote optimum immune function. 3) Clients with hepatitis need increased caloric intake so this is a good statement. 4)The client needs to understand some types of cough syrup have alcohol and all alcohol must be avoided to prevent further injury to the liver; therefore, this statement requires intervention"

A female client who has just been diagnosed with hepatitis A asks, "How could I have gotten this disease?" What is the nurse's best response? a. "You may have eaten contaminated restaurant food." b. "You could have gotten it by using I.V. drugs." c. "You must have received an infected blood transfusion." d. "You probably got it by engaging in unprotected sex.""

Answer A. Hepatitis A virus typically is transmitted by the oral-fecal route — commonly by consuming food contaminated by infected food handlers. The virus isn't transmitted by the I.V. route, blood transfusions, or unprotected sex. Hepatitis B can be transmitted by I.V. drug use or blood transfusion. Hepatitis C can be transmitted by unprotected sex."

Nurse Jaja is giving an injection to Ms. X. After giving an injection, the nurse accidentally stuck her finger with the needle when the client became very agitated. To determine if the nurse became infected with HIV when is the best time to test her for HIV antibodies? a. Immediately and repeat the test after 12 weeks b. Immediately and repeat the test after 4 weeks c. After a week and repeat the test in 4 months d. After a weeks and repeat the test in 6 months

Answer A. Keyword: BEST TIME. Rationale: To determine if a preexisting infection is present a test should be done immediately and is repeated again in 3 months time (12 weeks) to detect seroconversion as a result of the needle stick.

A client is admitted to the hospital with viral hepatitis, complaining of "no appetite" and "losing my taste for food." What instruction should the nurse give the client to provide adequate nutrition? a. Select foods high in fat b. Increase intake of fluids, including juices. c. Eat a good supper when anorexia is not as severe. d. Eat less often, preferably only three large meals daily."

Answer B : Although no special diet is required to treat viral hepatitis, it is generally recommended that clients consume a low-fat diet because fat may be tolerated poorly because of decreased bile production. Small frequent meals are preferable and may even prevent nausea. Frequently, appetite is better in the morining, so it is easier to eat a good breakfast. An adequated fluid intake of 2500 to 3000 mL/day that includes nutritional juices is also important.

The nurse observes precaution in caring for Mr. X as HIV is most easily transmitted in: a. Vaginal secretions and urine b. Breast milk and tears c. Feces and saliva d. Blood and semen

Answer D. Keyword: MOST EASILY. Rationale: HIV is MOST EASILY transmitted in blood, semen and vaginal secretions. However, it has been noted to be found in fecal materials, urine, saliva, tears and breast milk.

A client who is recently diagnosed with amyotrophic lateral sclerosis (ALS) is to be discharged home. The nurse should teach the family to: 1. monitor the client's urine output at home. 2. check the client's level of consciousness frequently. 3. check bony prominences and reposition the client every 2 hours. 4. serve the client a high red meat diet.

Answer: 3 1. It is not necessary to monitor the client's urine output, but the family is encouraged to provide good hydration. 2. Clients with ALS maintain their mental acuity. 3. The client is usually bedridden, and should be turned every 2 hours and monitored for breakdown on bony prominences. 4. Clients with ALS lose their ability to swallow, and could aspirate on pieces of meat.

A patient scheduled for a transurethral resection of the prostate (TURP) for BPH tells the nurse that he has delayed having surgery because he is afraid it will affect his sexual function. When responding to his concern, the nurse explains that a. with this type of surgery, erectile problems are rare, but retrograde ejaculation may occur. b. information about penile implants used for ED is available if he is interested. c. there are many methods of sexual expression that can be alternatives to sexual intercourse. d. sterility will not be a problem after surgery because sperm production will not be affected.

Answer: A Rationale: Erectile problems are rare, but retrograde ejaculation may occur after TURP. Erectile function is not usually affected by a TURP, so the patient will not need information about penile implants or reassurance that other forms of sexual expression may be used. Because the patient has not asked about fertility, reassurance about sperm production does not address his concerns.

The client enters the outpatient clinic and states to the triage nurse, "I think I have the flu. I'm so tired, I have no appetite, and everything hurts." The triage nurse assesses the client and finds a butterfly rash over the bridge of nose and on the cheeks. Which diagnosis does the nurse expect? A) Systemic lupus erythematosus B) Influenza type A C) Pnemonia D) Shingles

Answer: A The rash over the nose and cheeks is sometimes called a butterfly rash and is classic for the diagnosis of systemic lupus erythematosus (SLE), although not every client diagnosed with this disorder will have this rash.

You are administering erythropoietin to the patient with CKF. Which of the following would be a sign of adverse reaction? SATA A) Seizure B) Hypertension C) Decreased u/o D) Improved exercise tolerance E) Headache

Answer: A, B, and E. Seizures, Hypertension, HA, arthralgia, nausea, increased clotting of vascular access sites, seizures, and depletion of body iron stores are adverse effects of administering erythropoietin. Decreased u/o is a symptom of the disease process. Improved exercise tolerance would be a benefit of this medication.

The nurse knows that which of the following people groups are more likely to experience nontraditional signs and symptoms of a heart attack/angina? A) Elderly B) Patients with hx of heart failure C) Patients with hx of diabetes D) Women E) Patients taking a beta blocker

Answer: A, C, and D. The elderly, there are often no symptoms that present (silent CAD) and sometimes may have weakness or changes in mental status as signs of an MI. Patients with a history of diabetes also can have silent heart attacks. Women often experience atypical signs and symptoms of a heart attack such as fatigue, shoulder blade discomfort, and SOB.

A nurse is caring for a client with systemic lupus erythematosus (SLE). The client begins to cry and tells the nurse that she is afraid that her skin will be disfigured with lesions. Which intervention does the nurse plan to teach this client to minimize skin infections associated with SLE? Select all that apply. A) Use sunscreen with an SPF of 15 or greater. B) Remain indoors on sunny days. C) Avoid swimming in a pool or the ocean. D) Avoid sun exposure between 10:00 a.m. and 3:00 p.m. E) Decrease sun exposure between 3:00 p.m. and 5:00 p.m.

Answer: A, D The nurse teaches the client to live a normal life with a few extra precautions. There is a relationship between sun exposure and infection, so the client is taught to use sunscreen with an SPF of at least 15 and to avoid the sun between 10:00 a.m. and 3:00 p.m. The client may swim but should reapply sunscreen after swimming. The client does not need to stay indoors on sunny days or to decrease sun exposure between 3:00 p.m. and 5:00 p.m.

Which of the following would the nurse include as a priority when planning teaching for the patient diagnosed with ALS? A) Advanced Directives B) Bowel and bladder diversions C) Vision protection D) Treatment for dyskinesia

Answer: A. Advance directives would be the most important teaching point including discussion about ventilation and enteral nutrition. All are not closely related to ALS.

The wife of a patient who has undergone a TURP and has continuous bladder irrigation asks the nurse about the purpose of the continuous bladder irrigation. Which response by the nurse is appropriate? a. "The bladder irrigation is needed to stop the postoperative bleeding in the bladder." b. "The irrigation is needed to keep the catheter from being occluded by blood clots." c. "Normal production of urine is maintained with the irrigations until healing occurs." d. "Antibiotics are being administered into the bladder with the irrigation solution."

Answer: B Rationale: The purpose of bladder irrigation is to remove clots from the bladder and to prevent obstruction of the catheter by clots. The irrigation does not decrease bleeding or maintain urine production. Antibiotics are given by the IV route, not through the bladder irrigation.

The patient with ESRD arrives to the clinic ready for his peritoneal dialysis. He says "I am not very happy about being here today" This patient has a history of severe hypertension, heart failure, pulmonary edema, diabetes, A-fib, hyperlipidemia, CAD and has recently been diagnosed with osteoporosis. His vitals today are BP 145/70, HR 99, T 99.7 O2 94%. Which piece of patient data does the nurse need to pay most attention to right before beginning dialysis? A) The patient's anxiety B) Hx of diabetes C) BP 145/70 D) Hx of A-fib

Answer: B. It is important for the nurse to closely monitor the patient's glucose level because peritoneal dialysis uses solutions containing glucose. Insulin will probably need to be administered

The client is admitted with Parkinson's disease. His face is expressionless and his speech is monotone. Which of the following observations is the most accurate? A) The client is mostly likely depressed and should be left alone B) These are common symptoms of Parkinson's disease that produce an undesired façade of an alert and responsive individual C) The client's antipsychotic medications should be adjusted D) The client probably has dementia

Answer: B. The nurse should recognize that these are commons symptoms of Parkinson's disease.

Nurse Heather is looking over the patient chart and is preparing to administer erythropoietin to the patient with CKF. Which of the following pieces of information in the chart would cause Nurse Heather to question this order? A) Hgb of 9 B) Hx of uncontrolled HTN C) Pt. complains of fatigue D) Ferric Gluconate (Ferrlecit) is also ordered

Answer: B. Erythropoietin is used to treat anemia associated with chronic kidney disease. Uncontrolled HTN is a contraindication to this therapy because erythropoietin can severely raise BP. HGB of 9 is considered to be low, and would be an indication for the use of this medication. The HGB should not exceed 12. Fatigue is treated with this medication. Iron supplementation, ferric gluconate, is often ordered alongside erythropoietin to provide an adequate response.

A nurse is caring for a client with systemic lupus erythematous (SLE) who is taking hydroxychloroquine (Plaquenil). The nurse understands that the primary concern with this drug is: A) Pulmonary fibrosis. B) Cushingoid effects. C) Retinal toxicity. D) Renal toxicity.

Answer: C Hydroxychloroquine (Plaquenil) is an antimalarial drug used in SLE to reduce the frequency of acute episodes of SLE. The primary concern with Plaquenil is retinal toxicity and possible irreversible blindness. Cushingoid effects are a concern with corticosteroid therapy. Pulmonary fibrosis is a potential adverse effect of cyclophosphamide, not Plaquenil. Renal toxicity is not the primary concern with Plaquenil.

Which of the following is used to diagnose ALS? A) CT scan B) PET scan C) Physical Exam D) Lumbar puncture

Answer: C. The diagnosis of ALS is found based on clinical symptoms. An MRI and EMG may show some signs of the disease, they are not used to diagnose.

The patient undergoing peritoneal dialysis complains of abdominal pain. The nurse notes the drainage to be cloudy. She also palpates rebound tenderness. Which complication does the nurse suspect? A) Leakage around catheter B) Internal Bleeding C) Hypertriglycerdemia D) Peritonitis

Answer: D. Peritonitis is the most serious complication of PD. It's symtpoms include rebound tendernece, cloudy drainage, low grade fever, abdominal pain, and rebound tenderness.

The nurse is instructing the patient with stable angina about the use of nitroglycerin. Which of the following statements, made by the patient, indicates the need for further teaching? A) "When the pain is really bad, I can crush the tablet with my teeth to make it work faster" B) "I will be sure to never remove the pills from the container they are in" C) "It's probably a good idea to take the medication before I have sex with my wife" D) "I should call my doctor if I experience flushing of the face or a HA"

Answer: D. These are common side effects of this medication and there is not a need to call the physician for this issue. The patient can crush the tablet between his teeth, then place sublingually for hastening of effect. The nitroglycerin pills must be stored in a dry, cool place and should be kept in the dark bottles they come in. Nitroglycerin should be taken in advance of any activity that may produce pain, such as sexual activity.

The nurse is taking care of the patient with chronic kidney disease. Which of the following meal trays would be the best for this patient? A) Whole grain roll with baked chicken and pea soup and milk B) Sandwich with smoked salmon lunchmeat, green beans, and banana pudding C) Baked ham, mashed potatoes, tomato soup and peanut butter cookies D) Low-sodium chicken noodle soup, apple slices, white-wheat roll, and rice

Answer: D. This tray contains a small amount of protein and an adequate amount of carbohydrates that are low in sodium and potassium. Although a whole-grain roll would be appropriate with baked chicken. Pea soup and the milk would be high in potassium and protein. Smoked meats are often high in sodium. Tomato soup and peanut butter would add extra potassium and protein to this patient's diet

Which of the following clients is at the highest risk for experiencing an MI? a) The 71 year old African American female who smoked 10 years ago b) The 57 year old Caucasian male whose HDL is 30 and triglyceride level is 160 c) The 60 year old Native American male who has had an MI in the past d) The 65 year old Hispanic male with type II diabetes and peripheral arterial disease

Answer: D. Diabetes and PAD are considered CAD risk equivalents, meaning that they place the client at the same risk of experiencing a cardiac event as someone who has already had an MI. Considering this patients age, race, and two CAD risk equivalents, the nurse knows that this patient is at the highest risk for experiencing an MI.

Which client problem relating to altered nutrition is a consequence of AIDS? A. Increased appetite B. Decreased protein absorption C. Increased secretions of digestive juices D. Decreased gastrointestinal absorption

B Often the complications of the acquired immunodeficiency syndrome (AIDS) have a negative impact on nutritional status. Weight loss and protein depletion are commonly seen among the AIDS population.

The nurse is caring for a 55-yr-old man patient with acute pancreatitis resulting from gallstones. Which clinical manifestation would the nurse expect? A) Hematochezia B) Left upper abdominal pain C) Ascites and peripheral edema D) Temperature over 102 F

B Rationale: Abdominal pain (usually in the left upper quadrant) is the predominant manifestation of acute pancreatitis. Other manifestations of acute pancreatitis include nausea and vomiting, low-grade fever, leukocytosis, hypotension, tachycardia, and jaundice. Abdominal tenderness with muscle guarding is common. Bowel sounds may be decreased or absent. Ileus may occur and causes marked abdominal distention. Areas of cyanosis or greenish to yellow-brown discoloration of the abdominal wall may occur. Other areas of ecchymoses are the flanks (Grey Turner's spots or sign, a bluish flank discoloration) and the periumbilical area (Cullen's sign, a bluish periumbilical discoloration).

The nurse is conducting a physical assessment on a client with anemia. Which of the following clinical manifestations would be most indicative of the anemia? a. BP 146/88 b. Respirations 28 shallow c. Weight gain of 10 pounds in 6 months d. Pink complexion

B When there are fewer red blood cells, there is less hemoglobin and less oxygen. Therefore, the client is often short of breath. The client with anemia is often pale in color, has weight loss, and may be hypotensive.

To prevent the spread of hepatitis A virus, the nurse is especially careful when A. Disposing of food trays B. Emptying bed pans C. Taking an oral temperature D. Changing IV

B HAV is transmitted primarily person-to-person by the fecal-oral route. Food can be a method of transmission but needs to be fecally contaminated. Since the transmission of hepatitis A is fecal-oral and not saliva or blood like hepatitis B, taking a temperature and changing IV tubing would not spread hepatitis A.

When caring for a client with a left forearm arteriovenous fistula created for hemodialysis, which actions must the nurse take? (Select all that apply.) A Check brachial pulses daily. B Auscultate for a bruit every 8 hours. Correct C Teach the client to palpate for a thrill over the site. Correct D Elevate the arm above heart level. E Ensure that no blood pressures are taken in that arm.

B Auscultate for a bruit every 8 hours. C Teach the client to palpate for a thrill over the site. E Ensure that no blood pressures are taken in that arm. A bruit or swishing sound, and a thrill or buzzing sensation upon palpation should be present in this client, indicating patency of the fistula. No blood pressure, venipuncture, or compression, such as lying on the fistula, should occur. Distal pulses and capillary refill should be checked, and for a forearm fistula, the radial pulse is checked—the brachial pulse is proximal. Elevating the arm increases venous return, possibly collapsing the fistula.

A client has a platelet count of 9800/mm3. What action by the nurse is most appropriate? a. Assess the client for calf pain, warmth, and redness. b. Instruct the client to call for help to get out of bed. c. Obtain cultures as per the facility's standing policy. d. Place the client on protective isolation precautions.

B ~ A client with a platelet count this low is at high risk for serious bleeding episodes. To prevent injury, the client should be instructed to call for help prior to getting out of bed. Calf pain, warmth, and redness might indicate a deep vein thrombosis, not associated with low platelets. Cultures and isolation relate to low white cell counts.

A client has received a dose of ondansetron (Zofran) for nausea. What action by the nurse is most important? a. Assess the client for a headache. b. Assist the client in getting out of bed. c. Instruct the client to reduce salt intake. d. Weigh the client daily before the client eats.

B ~ Ondansetron side effects include postural hypotension, vertigo, and bradycardia, all of which increase the clients risk for injury. The nurse should assist the client when getting out of bed. Headache and fluid retention are not side effects of this drug.

What is the first intervention for a client experiencing MI? A) Administer morphine B) Administer oxygen C) Administer sublingual nitroglycerin D) Obtain an ECG

B) Administer oxygen Administering supplemental oxygen to the client is the first priority of care. The myocardium is deprived of oxygen during an infarction, so additional oxygen is administered to assist in oxygenation and prevent further damage. Morphine and nitro are also used to treat MI, but they're more commonly administered after the oxygen. An ECG is the most common diagnostic tool used to evaluate MI

Aspirin is administered to the client experiencing an MI because of its: A) Antipyrectic action B) Antithrombotic action C) Antiplatelet action D) Analgesic action

B) Antithrombotic action Aspirin does have antipyretic, antiplatelet, and analgesic actions, but the primary reason ASA is administered to the client experiencing an MI is its antithrombotic action.

What does the nurse include in the teaching plan for a client receiving a beta blocker for treatment of angina? A) Discontinue drug if heart rate <60. B) Do not discontinue drug abruptly. C) Exercise heart rate should be 110-120. D) Monitor for hyperglycemia.

B) Do not discontinue drug abruptly. Rationale: Beta blocker treatment should never be abruptly discontinued. With abrupt cessation, a rebound excitation occurs, and adrenergic receptors are stimulated. This can exacerbate angina, increase heart rate, and cause myocardial infarction. Clients often tolerate heart rates as low as 50. The beta blocker might blunt the compensatory increase in heart rate with exercise. Hypoglycemia can occur.

The nurse is reviewing the use of multidrug therapy for HIV with a patient. Which statements are correct regarding the reason for using multiple drugs to treat HIV? (Select all that apply.) a. The combination of drugs has fewer associated toxicities. b. The use of multiple drugs is more effective against resistant strains of HIV. c. Effective treatment results in reduced T-cell counts. d. The goal of this treatment is to reduce the viral load. e. This type of therapy reduces the incidence of opportunistic infections.

B, D, E

Major measures to help prevent cancer in patients should be taught by the nurse at every given opportunity. What are those things that should be taught to patients about the prevention of cancer? (Select all that apply.) A. Increase alcohol intake. B. Maintain a normal weight. C. Decrease ascorbic acid in the diet. D. Abstain from nitrite and nitrate food additives. E. Have your drinking water supply checked for contaminants.

B, D, E Major measures to help prevent cancer in patients should be taught by the nurse at every given opportunity. Those things that should be taught to patients about the prevention of cancer include moderation in drinking alcohol, increasing (not decreasing) ascorbic acid (vitamin C), maintenance of normal weight, checking the chemical makeup of the local water supply, and avoiding nitrite and nitrate food additives.

A patient with sudden pain in the left upper quadrant radiating to the back and vomiting was diagnosed with acute pancreatitis. Which intervention should the nurse include in the patient's plan of care? A) Immediately start enteral feeding to prevent malnutrition. B) Insert an NG and maintain NPO status to allow pancreas to rest. C) Initiate early prophylactic antibiotic therapy to prevent infection. D) Administer acetaminophen (Tylenol) every 4 hours for pain relief.

B. Rationale: Initial treatment with acute pancreatitis will include an NG tube if there is vomiting and being NPO to decrease pancreatic enzyme stimulation and allow the pancreas to rest and heal. Fluid will be administered to treat or prevent shock. The pain will be treated with IV morphine because of the NPO status. Enteral feedings will only be used for the patient with severe acute pancreatitis in whom oral intake is not resumed. Antibiotic therapy is only needed with acute necrotizing pancreatitis and signs of infection.

Which client statement regarding his or her diagnosis of HIV infection indicates a need that further teaching is necessary? A. "I must take these medications exactly as prescribed for the rest of my life." B. "I don't need to use condoms as long as I take my medication as prescribed." C. "I will notify my health care provider immediately if I bruise or bleed more easily than normal." D. "I should remain upright for 30 minutes after taking my zidovudine to prevent esophageal ulceration."

B. "I don't need to use condoms as long as I take my medication as prescribed." Antiretroviral drugs do not stop the transmission of HIV, and clients need to continue standard precautions and safe sex practice, including condom use. Potential serious adverse effects of zidovudine are bone marrow suppression and esophageal ulceration.

The nurse determines that the patient understood medication instructions about the use of a spacer device when taking inhaled medications after hearing the patient state what as the primary benefit? A. "I will pay less for medication because it will last longer." B. "More of the medication will get down into my lungs to help my breathing." C. "Now I will not need to breathe in as deeply when taking the inhaler medications." D. "This device will make it so much easier and faster to take my inhaled medications."

B. "More of the medication will get down into my lungs to help my breathing." A spacer assists more medication to reach the lungs, with less being deposited in the mouth and the back of the throat. It does not affect the cost or increase the speed of using the inhaler.

A patient receiving propylthiouracil (PTU) asks the nurse, "How does this medication relieve symptoms?" What is the nurse's best response? A. "PTU helps your thyroid gland synthesize and use iodine, which produces hormones better." B. "PTU inhibits the formation of new thyroid hormone, thus returning your metabolism to normal." C. "PTU causes the pituitary gland to secrete thyroid-stimulating hormone, which blocks the production of hormones by the thyroid gland." D. "PTU removes thyroid hormones that are already circulating in your bloodstream, thus decreasing the adverse effects of this medication."

B. "PTU inhibits the formation of new thyroid hormone, thus returning your metabolism to normal." PTU is an antithyroid medication used to treat hyperthyroidism. It works by inhibiting the synthesis of new thyroid hormone. It does not inactivate present hormone.

The nurse administers intravenous mannitol for an unconscious client. A decrease in which of the following is expected as a therapeutic effect of this drug? A. Seizure activity B. Cerebral edema C. Cerebral metabolism D. Cerebral inflammation

B. Cerebral edema

The nurse was accidently stuck with a needle used on an HIV-positive patient. After reporting this, what care should this nurse first receive? A. Personal protective equipment B. Combination antiretroviral therapy C. Counseling to report blood exposures D. A negative evaluation by the manage

B. Combination antiretroviral therapy Postexposure prophylaxis with combination antiretroviral therapy can significantly decrease the risk of infection. Personal protective equipment should be available although it may not have stopped this needle stick. The needle stick has been reported. The negative evaluation may or may not be needed but would not occur first.

The nurse notes that the respiratory symptoms of the patient with chronic obstructive pulmonary disease (COPD) have affected his nutrition. Which would most help improve the patient's nutrition? A. Eat three large meals to increase stomach fullness. B. Extra protein is required to repair damaged tissues. C. Exercise before eating each meal three times a day. D. Drink six to eight glasses of caffeinated fluids each day.

B. Extra protein is required to repair damaged tissues. Nutrition is very important for the patient with COPD because the extra work to breathe properly uses more calories and anorexia may be present. Extra protein is required to repair damaged tissues. It is helpful to drink six to eight glasses of noncaffeinated fluids per day to keep mucus thin and easier to cough up, unless the physician has the patient on a fluid restriction. The patient should rest before eating, not exercise before meals. The patient should avoid overeating and should eat four to six small meals a day rather than three regular meals to decrease stomach fullness and reduce fatigue.

The LPN/LVN should understand that invasive procedures are postponed for 4 to 6 hours after hemodialysis for which reason? A. The procedure is very tiring. B. Heparin is used during the procedure. C. The patient is disoriented immediately after the procedure. D. The chance of infection is heightened immediately after the procedure.

B. Heparin is used during the procedure. Invasive procedures are postponed for 4 to 6 hours after dialysis because the clotting time is extended from the heparin used during dialysis and prolonged bleeding could occur. Disorientation of the patient after the procedure is not anticipated. Although the patient may be fatigued after the procedure, this is not the most important reason for avoiding invasive procedures in the period of time immediately after hemodialysis. Infection risk is not increased after dialysis.

Anemia or insufficient hemoglobin content is common in older persons. The client's body compensates for the deficiency by: A. Decreasing the respiratory and heart rates. B. Increasing the heart and respiratory rates. C. Shunting blood away from vital organs and skin. D. Decreasing blood viscosity in order to supply oxygen to hypoxic tissues.

B. Increase the Heart & RR all anemias result in loss of oxygen carrying capacity of the blood and generalized hypoxia. The body compensates for this by raising HR and RR

For a patient taking levothyroxine (Synthroid) and warfarin (Coumadin) concurrently, the nurse would closely monitor for which possible serious adverse effect? A. Acute confusion B. Increased bruising C. Cardiac dysrhythmias D. Orthostatic hypotension

B. Increased bruising Levothyroxine can compete with protein-binding sites of warfarin, allowing more warfarin to be unbound or free, thus increasing effects of warfarin and risk of bleeding. Bleeding commonly presents as bruising.

The patient with sudden pain in the left upper quadrant radiating to the back and vomiting was diagnosed with acute pancreatitis. What intervention(s) should the nurse expect to include in the patient's plan of care? A. Immediately start enteral feeding to prevent malnutrition. B. Insert an NG and maintain NPO status to allow pancreas to rest. C. Initiate early prophylactic antibiotic therapy to prevent infection. D. Administer acetaminophen (Tylenol) every 4 hours for pain relief.

B. Insert an NG and maintain NPO status to allow pancreas to rest. Initial treatment with acute pancreatitis will include an NG tube if there is vomiting and being NPO to decrease pancreatic enzyme stimulation and allow the pancreas to rest and heal. Fluid will be administered to treat or prevent shock. The pain will be treated with IV morphine because of the NPO status. Enteral feedings will only be used for the patient with severe acute pancreatitis in whom oral intake is not resumed. Antibiotic therapy is only needed with acute necrotizing pancreatitis and signs of infection.

The nurse would suspect a patient is taking too much levothyroxine (Synthroid) when the patient exhibits which adverse effect? A. Lethargy B. Irritability C. Feeling cold D. Weight gain

B. Irritability Irritability is a symptom of hyperthyroidism and may indicate toxicity of the medication. The other choices are signs of hypothyroidism.

A client has fluid leaking from the nose after a basilar skull fracture. Which of the following would indicate that the fluid is cerebrospinal fluid? A. It clumps together on the paper and has a pH of 7 B. It leaves a yellowish ring on the paper and tests positive for glucose. C. It is grossly bloody in appearance and has a pH of 6. D. It is clear in appearance and tests negative for glucose.

B. It leaves a yellowish ring on the paper and tests positive for glucose. This is often called the "halo" sign

The nurse is caring for a 55-year-old man patient with acute pancreatitis resulting from gallstones. Which clinical manifestation would the nurse expect the patient to exhibit? A. Hematochezia B. Left upper abdominal pain C. Ascites and peripheral edema D. Temperature over 102o F (38.9o C)

B. Left upper abdominal pain Abdominal pain (usually in the left upper quadrant) is the predominant manifestation of acute pancreatitis. Other manifestations of acute pancreatitis include nausea and vomiting, low-grade fever, leukocytosis, hypotension, tachycardia, and jaundice. Abdominal tenderness with muscle guarding is common. Bowel sounds may be decreased or absent. Ileus may occur and causes marked abdominal distention. Areas of cyanosis or greenish to yellow-brown discoloration of the abdominal wall may occur. Other areas of ecchymoses are the flanks (Grey Turner's spots or sign, a bluish flank discoloration) and the periumbilical area (Cullen's sign, a bluish periumbilical discoloration).

A patient was recently discharged home for treatment of hypothyroidism and was ordered to take Synthroid for treatment. The patient is re-admitted with signs and symptoms of the following: heart rate 42, blood pressure 70/56, blood glucose 55, and body temperature of 96.8 'F. The patient is very fatigued and drowsy. The family reports the patient has not been taking Synthroid since being discharged home from the hospital. Which of the following conditions is this patient most likely experiencing? A. Thryoid Storm B. Myxedema Coma C. Iodism D. Toxic Nodular Goiter

B. Myxedema Coma The red flags in this question are the patient's signs/symptoms and the report from the family the patient hasn't been taking the prescribed Synthroid. The patient is showing signs and symptoms of extreme hypothyroidism known as Myxedema coma (which is life-threatening if not treated).

Which medication does the nurse plan to administer to a client before chemotherapy to decrease the incidence of nausea? A. Morphine B. Ondansetron (Zofran) C. Naloxone (Narcan) D. Diazepam (Valium)

B. Ondansetron (Zofran) Rationale A. Morphine is a narcotic analgesic or opiate; it may cause nausea. B. Ondansetron is a 5-HT3 receptor blocker that blocks serotonin to prevent nausea and vomiting. C. Naloxone is a narcotic antagonist used for opiate overdose. D. Diazepam is a benzodiazepine, which is an antianxiety medication only. Lorazepam, a benzodiazepine, may be used for nausea.

The nurse determines that a patient is experiencing common adverse effects from the inhaled corticosteroid fluticasone after what occurs? A. Hypertension and pulmonary edema B. Oropharyngeal candidiasis and hoarseness C. Elevation of blood glucose and calcium levels D. Adrenocortical dysfunction and hyperglycemia

B. Oropharyngeal candidiasis and hoarseness Oropharyngeal candidiasis and hoarseness are common adverse effects from the use of inhaled corticosteroids because the medication can lead to overgrowth of organisms and local irritation if the patient does not rinse the mouth following each dose.

An 86 year old female patient is immobile and is in the right lateral recumbent position. As the nurse you know that which sites below are at most risk for pressure injury in this position? A. Sacral B. Patella C. Ankle D. Ear E. Elbow F. Hip G. Heel H. Shoulder

B. Patella C. Ankle D. Ear F. Hip H. Shoulder The right lateral recumbent position is where the patient is positioned on their right side. Therefore, the ankle, ear, hip, knee, and shoulders are sites where a pressure injury can occur.

What are the main advantages of peritoneal dialysis compared to hemodialysis? A. No medications are required because of the enhanced efficiency of the peritoneal membrane in removing toxins. B. The diet is less restricted and dialysis can be performed at home. C. The dialysate is biocompatible and causes no long-term consequences. D. High glucose concentrations of the dialysate cause a reduction in appetite, promoting weight loss.

B. The diet is less restricted and dialysis can be performed at home. Advantages of peritoneal dialysis include fewer dietary restrictions and home dialysis is possible.

Which of the following are common sites for development of pressure ulcers? (select all that apply) A. sternum B. heels C. sacrum D. ears E. lateral malleoli F. trochanters G. tip of great toe

B. heels C. sacrum D. ears E. lateral maleoli F. trochanters

After giving an injection to a patient with HIV infection, the nurse accidentally receives a needlestick from a too-full needle disposal box. Recommendations for occupational HIV exposure may include the use of which drug(s)? a. didanosine b. lamivudine and enfuvirtide c. emtricitabine and tenofovir d. acyclovir

C

The client diagnosed with acute pancreatitis is in pain. Which position should the nurse assist the client to assume to help decrease the pain? A. Recommend lying in the prone position with legs extended B. Maintain a tripod position over the bedside table C. Place in side-lying position with knees flexed D. Encourage a supine position with a pillow under the knees

C Rationale: The fetal position deceases pain caused by the stretching of the peritoneum as a result of edema. The pancreas is located abdomen. Anything that causes the abdomen to be stretched will increase pain.

A client with end-stage kidney disease has been put on fluid restrictions. Which assessment finding indicates that the client has not adhered to this restriction? A Blood pressure of 118/78 mm Hg B Weight loss of 3 pounds during hospitalization C Dyspnea and anxiety at rest D Central venous pressure (CVP) of 6 mm Hg

C Dyspnea and anxiety at rest Dyspnea is a sign of fluid overload and possible pulmonary edema; the nurse should assist the client in correlating symptoms of fluid overload with nonadherence to fluid restriction. Nonadherence to fluid restriction results in fluid volume excess and higher blood pressures; 118/78 mm Hg is a normal blood pressure. Excess fluid intake and fluid retention are manifested by an elevated CVP (>8 mm Hg) and weight gain, not weight loss.

Which foods will the nurse teach the client with chronic pancreatitis to avoid? Select all that apply. A ~ Blueberries B ~ Green beans C ~ Bacon D ~ Baked fish E ~ Fried potatoes

C ~ Bacon E ~ Fried potatoes These are fatty foods

A patient with a diagnosis of lung cancer is told that there is metastasis of the disease. Which statement by the patient indicates an understanding of his current condition? A ~ "Metastasis means that like cells have joined together into a mass." B ~ "Metastasis means that there is stabilization of a group of cells in one site." C ~ "Metastasis means that there is movement of cells from one part of my body to another part of my body." D ~ "Metastasis means that there has been an invasion of malignant cells into multiple normal body tissues simultaneously."

C ~ Metastasis occurs when cancer cells have moved from one part of the body to another site in the body.

The nurse determines that treatment of a client with a beta-adrenergic blocker for myocardial infarction has been effective when: A) Tachycardia occurs. B) Blood pressure is 90/50. C) Decreased dysrhythmias occur. D) Decreased urinary output occurs.

C) Decreased dysrhythmias occur. Rationale: Beta blockers have the ability to decrease heart rate, decrease contractility, and decrease blood pressure, leading to decreased oxygen demand. They also slow conduction, which suppresses dysrhythmias. Tachycardia would not be desired with an MI. A low BP alone would not indicate effective treatment of the MI.

A patient with multiple sclerosis (MS) is to begin treatment with glatiramer acetate (Copaxone). Which information will the nurse include in patient teaching? a. Recommendation to drink at least 3 to 4 L daily b. Need to avoid driving or operating heavy machinery c. How to draw up and administer injections of the medication d. Use of contraceptive methods other than oral contraceptives

C. Copaxone is administered by self-injection. Oral contraceptives are an appropriate choice for birth control. There is no need to avoid driving or drink large fluid volumes when taking glatiramer.

The client is admitted to the medical department with a diagnosis of acute pancreatitis. What laboratory values should the nurse monitor to confirm this diagnosis? A. Creatinine and BUN B. Troponin and CK-MB C. Serum amylase and lipase D. Serum bilirubin and calcium

C. Rationale: Serum amylase levels increase within two to 12 hours of the onset of acute pancreatitis; lipase elevates and remains elevated for seven to 14 days

After a patient experienced a brief episode of tinnitus, diplopia, and dysarthria with no residual effects, the nurse anticipates teaching the patient about a. cerebral aneurysm clipping. b. heparin intravenous infusion. c. oral low-dose aspirin therapy. d. tissue plasminogen activator (tPA).

C. The patient's symptoms are consistent with transient ischemic attack (TIA), and drugs that inhibit platelet aggregation are prescribed after a TIA to prevent stroke. Continuous heparin infusion is not routinely used after TIA or with acute ischemic stroke. The patient's symptoms are not consistent with a cerebral aneurysm. tPA is used only for acute ischemic stroke, not for TIA.

The nurse provides discharge instructions for a 64-year-old woman with ascites and peripheral edema related to cirrhosis. Which statement, if made by the patient, indicates teaching was effective? A. "It is safe to take acetaminophen up to four times a day for pain." B. "Lactulose (Cephulac) should be taken every day to prevent constipation." C. "Herbs and other spices should be used to season my foods instead of salt." D. "I will eat foods high in potassium while taking spironolactone (Aldactone)."

C. "Herbs and other spices should be used to season my foods instead of salt." A low-sodium diet is indicated for the patient with ascites and edema related to cirrhosis. Table salt is a well-known source of sodium and should be avoided. Alternatives to salt to season foods include the use of seasonings such as garlic, parsley, onion, lemon juice, and spices. Pain medications such as acetaminophen, aspirin, and ibuprofen should be avoided as these medications may be toxic to the liver. The patient should avoid potentially hepatotoxic over-the-counter drugs (e.g., acetaminophen) because the diseased liver is unable to metabolize these drugs. Spironolactone is a potassium-sparing diuretic. Lactulose results in the acidification of feces in bowel and trapping of ammonia, causing its elimination in feces.

Which teaching point does the nurse include for a client with peripheral arterial disease (PAD)? A. "Elevate your legs above heart level to prevent swelling." B. "Inspect your legs daily for brownish discoloration around the ankles." C. "Walk to the point of leg pain, then rest, resuming when pain stops." D. "Apply a heating pad to the legs if they feel cold."

C. "Walk to the point of leg pain, then rest, resuming when pain stops." Exercise may improve arterial blood flow by building collateral circulation; instruct the client to walk until the point of claudication, stop and rest, and then walk a little farther.

A patient has been admitted to the coronary care unit with a diagnosis of acute MI. The patient is experiencing severe chest pain. What intervention is MOST likely to relieve the patient's pain? A. Proper positioning B. Administration of codeine C. Administration of morphine D. Administration of beta blockers

C. Administration of morphine Pain control is a high priority for the patient experiencing an acute MI. Morphine is the drug of choice because of its vasodilation property. Proper positioning and administration of codeine or beta blockers are not the most appropriate interventions.

A patient who is to have a growth in her breast removed says to the LPN/LVN, "How are benign tumors different from cancerous ones?" Which response by the nurse is most accurate? A. "Benign tumor cells replicate faster than cancerous cells." B. "Benign tumor cells tend to spread easier than cancerous tumor cells." C. "Cancerous tumor cells have changes in the DNA." D. "Cancerous tumors are usually encapsulated."

C. Cancerous cells have changes to the DNA, replicate much faster than benign cells, and have the ability to spread to other tissues. Benign tumors are generally encapsulated.

A nurse is caring for a client admitted to the hospital with respiratory difficulty after being diagnosed with amyotrophic lateral sclerosis (ALS) approximately 1 year ago. Which of the following client finding should the nurse anticipate? Select all that apply. A. Loss of sensation B. Fluctuations in blood pressure C. Incontinence D. Ineffective cough E. Loss of cognitive function

C. D. A) Sensory changes are not associated with ALS B) Fluctuations in blood pressure are not a finding in ALS C) Incontince due to muscle weakness is a finding in ALS D) Ineffective cough due to progressive muscle weakness E) no rationale

A patient is admitted with complaints of palpations, excessive sweating, and unable to tolerate heat. In addition, the patient voices concern about how her appearance has changed over the past year. The patient presents with protruding eyeballs and pretibial myxedema on the legs and feet. Which of the following is the likely cause of the patient's signs and symptoms? A. Thyroiditis B. Deficiency of iodine consumption C. Grave's Disease D. Hypothyroidism

C. Grave's Disease

___________ is an autoimmune disorder where the body attacks the thyroid gland that causes it to stop releasing T3 and T4. The patient is likely to have the typical signs/symptoms of hypothyroidism, however, they may present with what other sign as well? A. Myxedema coma; joint pain B. Thyroid storm; memory loss C. Hashimoto's Thyroiditis; goiter D. Toxic nodular goiter (TNG); goiter

C. Hashimoto's Thyroiditis; goiter

The nurse assesses a patient with emphysema and notes a barrel chest. What is the reason for this patient's chest anomaly? A. Collapse of distal alveoli B. Use of accessory muscles C. Hyperinflation of the lungs D. Long-term, chronic hypoxia

C. Hyperinflation of the lungs A patient with emphysema develops a barrel chest as a result of the trapping of air in the lungs, which causes them to hyperinflate. Collapsed alveoli, use of accessory muscles, and long-term, chronic hypoxia do not cause a barrel chest.

A patient with angina is being discharged and has been prescribed nitroglycerin (NTG) tablets. What instructions should the nurse give this patient regarding the medication? A. It is necessary to refill the supply every 6 months. B. The tablet should be crushed and taken with water. C. Lie down when using NTG tablets. D. One NTG tablet should be taken every 15 minutes if pain occurs.

C. Lie down when using NTG tablets. The patient should lie down when taking NTG tablets. The expiration dates on the bottle should be checked frequently and the tablets should be replaced accordingly. One tablet should be taken every 5 minutes, up to three times. If the pain does not subside, the patient should then call for help and be taken to the emergency room. The tablets should be taken sublingually, not crushed or swallowed.

How should you assess the patency of a newly placed arteriovenous graft for dialysis? A. Irrigate the graft daily with low-dose heparin. B. Monitor for any increase in blood pressure in the affected arm. C. Listen with a stethoscope over the graft for presence of a bruit. D. Frequently monitor the pulses and neurovascular status distal to the graft.

C. Listen with a stethoscope over the graft for presence of a bruit. A thrill can be felt by palpating the area of anastomosis of the arteriovenous graft, and a bruit can be heard with a stethoscope. The bruit and thrill are created by arterial blood rushing into the vein.

The nurse is monitoring the client with ↑intracranial pressure (ICP). Which of the following does the nurse expect to be ordered to maintain the ICP within a specified range? A. Dexamethasone (Decadron) B. Hydrochlorothiazide (HydroDIURIL) C. Mannitol (Osmitrol) D. Phenytoin (Dilantin)

C. Mannitol (Osmitrol)

A client with peripheral arterial disease (PAD) has undergone percutaneous transluminal angioplasty (PTA) of the lower extremity. What is essential for the nurse to assess after the procedure? A. Ankle-brachial index B. Dye allergy C. Pedal pulses D. Gag reflex

C. Pedal pulses Priority nursing care focuses on assessment for bleeding at the arterial puncture site and monitoring for distal pulses. Pulse checks must be assessed postprocedure to detect improvement (stronger pulses) or complications (diminished or absent pulses).

The nurse is assessing the client with a traumatic brain injury after a skateboarding accident. Which symptom is the nurse most concerned about? A. Amnesia for events of accident B. Bleeding head laceration C. Pupil changes in one eye D. Restlessness and confusion

C. Pupil changes in one eye

Which of the following nursing diagnoses are most appropriate for the client who has an intraventricular catheter for intracranial pressure monitoring? A. Risk for fluid volume excess related to infusion into ventricle B. Risk for altered skin integrity related to need to remain in supine position C. Risk for infection related to catheter inserted into ventricle D. Pain related to wound in scalp for insertion of monitoring device

C. Risk for infection related to catheter inserted into ventricle

A patient is being discharged home for treatment of hypothyroidism. Which medication is most commonly prescribed for this condition? A. Tapazole B. PTU (Propylthiouracil) C. Synthroid D. Inderal

C. Synthroid is the only medication listed that treats hypothyroidism. All the other medications are used for hyperthyroidism.

For which reason are patients with esophageal varices prone to hemorrhage? A. They have portal hypotension. B. There is poor circulation within the veins. C. They are no longer able to produce vitamin K. D. There is an accumulation of ammonia in the blood.

C. They are no longer able to produce vitamin K. Esophageal varices are engorged veins (similar to varicose veins) that line the esophagus. They are the result of portal congestion and hypertension. The congestion can lead to massive bleeding when the vein walls rupture from increased pressure or esophageal irritation. Another factor in hemorrhage is that the liver is no longer able to make vitamin K. Ammonia buildup does not increase the patient's risk for hemorrhage.

The woman is afraid she may get HIV from her bisexual husband. What should the nurse include when teaching her about preexposure prophylaxis (select all that apply)? A. Take fluconazole (Diflucan). B. Take amphotericin B (Fungizone). C. Use condoms for risk-reducing sexual relations. D. Take emtricitabine and tenofovir (Truvada) regularly. E. Have regular HIV testing for herself and her husband.

C. Use condoms for risk-reducing sexual relations. D. Take emtricitabine and tenofovir (Truvada) regularly. E. Have regular HIV testing for herself and her husband. Using male or female condoms, having monthly HIV testing for the patient and her husband, and the woman taking emtricitabine and tenofovir regularly has shown to decrease the infection of heterosexual women having sex with a partner who participates in high-risk behavior. Fluconazole and amphotericin B are taken for Candida albicans, Coccidioides immitis, and Cryptococcosus neoformans, which are all opportunistic diseases associate with HIV infection

A client is admitted to ICU from PACU after craniotomy to remove a clot in the frontal lobe. How will the nurse position the client? A. With flexed knees to decrease intra-abdominal pressure B. On the right side to prevent bleeding at incision site C. With head of bed elevated at least 30 degrees to promote venous drainage D. Log rolled to bed and head of bed no more than 15 degrees elevation

C. With head of bed elevated at least 30 degrees to promote venous drainage

Measures indicated in the conservative therapy of CKD include A. decreased fluid intake, carbohydrate intake, and protein intake. B. increased fluid intake; decreased carbohydrate intake and protein intake. C. decreased fluid intake and protein intake; increased carbohydrate intake. D. decreased fluid intake and carbohydrate intake; increased protein intake.

C. decreased fluid intake and protein intake; increased carbohydrate intake. Water and any other fluids are not routinely restricted in the pre-end-stage renal disease (ESRD) stages. Patients on hemodialysis have a more restricted diet than patients receiving peritoneal dialysis. For those receiving hemodialysis, as their urinary output diminishes, fluid restrictions are enhanced. Intake depends on the daily urine output. Generally, 600 mL (from insensible loss) plus an amount equal to the previous day's urine output is allowed for a patient receiving hemodialysis. Patients are advised to limit fluid intake so that weight gains are no more than 1 to 3 kg between dialyses (interdialytic weight gain). For the patient who is undergoing dialysis, protein is not routinely restricted. The beneficial role of protein restriction in CKD stages 1 through 4 as a means to reduce the decline in kidney function is being studied. Historically, dietary counseling often encouraged restriction of protein for CKD patients. Although there is some evidence that protein restriction has benefits, many patients find these diets difficult to adhere to. For CKD stages 1 through 4, many clinicians encourage a diet with normal protein intake. However, you should teach patients to avoid high-protein diets and supplements because they may overstress the diseased kidneys.

What term refers to pale, red and watery drainage from a wound? A. serous B. sanguineous C. serosanguineous D. purulent

C. serosanguineous serous - clear, watery, plasma sanguineous - bright red, active bleeding purulent - thick, yellow, green, tan or brown (pus)

The nurse instructs a client diagnosed with hepatitis A about untoward signs and symptoms related to hepatitis that may develop. The one that should be reported to the practitioner is: 1)Fatigue 2)Anorexia 3)Yellow urine 4)Clay-covered stools

Correct 4 Clay-colored stools are indicative of hepatic obstruction because bile is prevented from entering the intestines.

A client has developed hepatitis A after eating contaminated oysters. The nurse assesses the client for which of the following? 1. Malaise 2. Dark stools 3. Weight gain 4. Left upper quadrant discomfort

Correct Answer 1 Rationale: Hepatitis causes GI symptoms such as anorexia, nausea, right upper quadrant discomfort and weight loss. Fatigue and malaise are common. Stools will be light or clay colored if conjugated bilirubin is unable to flow out of the liver because of inflammation or obstruction of the bile ducts.

A client is suspected of having hepatitis. Which diagnostic test result will assist in confirming this diagnosis ? A.Elevate hemoglobin level B. Elevated serum bilirubin level C. Elevated blood urea nitrogen level D. Decreased erythrocyte sedimentation rate

Correct Answer B Laboratory indicator of hepatitis include elevated liver enzyme levels, elevated serum bilirubin levels.Thinking about the organ that is involved in hepatitis should assist in directing to choose option B liver function test.

What type of precaution should the nurse implement to protect from being exposed to any of the hepatitis viruses? A. Airborne Precautions. B. Standard Precautions. C. Droplet Precautions. D. Exposure Precautions.

Correct Answer B: Standard precautions apply to blood, all body fluids, secretions, and excretions, except sweat, regardless of whether they contain visible blood. Airborne Precautions are only for airborne droplet nuclei or dust particles, Droplet precaution involves large particle droplets in the mucus membranes, and Exposure precaution is not a designated isolation category.

Which of the following signs and symptoms in a patient with a T4 spinal cord injury should alert the nurse to the possibility of autonomic dysreflexia? A) Headache and rising blood pressure B) Irregular respirations and shortness of breath C) Decreased level of consciousness or hallucinations D) Abdominal distention and absence of bowel sounds

Correct Answer(s): A Among the manifestations of autonomic dysreflexia are hypertension (up to 300 mm Hg systolic) and a throbbing headache. Respiratory manifestations, decreased level of consciousness, and gastrointestinal manifestations are not characteristic.

Which of the following clinical manifestations would the nurse interpret as representing neurogenic shock in a patient with acute spinal cord injury? A) Bradycardia B) Hypertension C) Neurogenic spasticity D) Bounding pedal pulses

Correct Answer(s): A Neurogenic shock is due to the loss of vasomotor tone caused by injury and is characterized by hypotension and bradycardia. Loss of sympathetic innervation causes peripheral vasodilation, venous pooling, and a decreased cardiac output.

The nurse is caring for a patient admitted with a spinal cord injury following a motor vehicle accident. The patient exhibits a complete loss of motor, sensory, and reflex activity below the injury level. The nurse recognizes this condition as which of the following? A) Central cord syndrome B) Spinal shock syndrome C) Anterior cord syndrome D) Brown-Séquard

Correct Answer(s): B About 50% of people with acute spinal cord injury experience a temporary loss of reflexes, sensation, and motor activity that is known as spinal shock. Central cord syndrome is manifested by motor and sensory loss greater in the upper extremities than the lower extremities. Anterior cord syndrome results in motor and sensory loss but not reflexes. Brown-Séquard syndrome is characterized by ipsilateral loss of motor function and contralateral loss of sensory function.

A patient with a history of a T2 spinal cord tells the nurse, "I feel awful today. My head is throbbing, and I feel sick to my stomach." Which action should the nurse take first? a. Notify the patient's health care provider. b. Check the blood pressure (BP). c. Give the ordered antiemetic. d. Assess for a fecal impaction.

Correct Answer(s): B Rationale: The BP should be assessed immediately in a patient with an injury at the T6 level or higher who complains of a headache to determine whether autonomic dysreflexia is causing the symptoms, including hypertension. Notification of the patient's health care provider is appropriate after the BP is obtained. Administration of an antiemetic is indicated after autonomic dysreflexia is ruled out as the cause of the nausea. The nurse may assess for a fecal impaction, but this should be done after checking the BP and lidocaine jelly should be used to prevent further increases in the BP.

The nurse administers methylprenisone(Solu-Medrol) as a continuous IV fusion to a male patient who has fractures of the cervical vertebrae. Which intervention would prevent or detect adverse effects of the medication? A. record pt baseline weight B. administer PPI( proton pump inhibitor) C. Check the hear rate for bradycardia D. suction the patient's oropharynx

Correct Answer(s): B the nurse should administer PPI because they are at high risk for GI erosion and bleeding from the steroid.

When caring for a patient who was admitted 24 hours previously with a C5 spinal cord injury, which nursing action has the highest priority? a. Continuous cardiac monitoring for bradycardia b. Administration of methylprednisolone (Solu-Medrol) infusion c. Assessment of respiratory rate and depth d. Application of pneumatic compression devices to both legs

Correct Answer(s): C Rationale: Edema around the area of injury may lead to damage above the C4 level, so the highest priority is assessment of the patient's respiratory function. The other actions are also appropriate but are not as important as assessment of respiratory effort.

The health care provider orders administration of IV methylprednisolone (Solu-Medrol) for the first 24 hours to a patient who experienced a spinal cord injury at the T10 level 3 hours ago. When evaluating the effectiveness of the medication the nurse will assess a. blood pressure and heart rate. b. respiratory effort and O2 saturation. c. motor and sensory function of the legs. d. bowel sounds and abdominal distension.

Correct Answer(s): C Rationale: The purpose of methylprednisolone administration is to help preserve neurologic function; therefore, the nurse will assess this patient for lower-extremity function. Sympathetic nervous system dysfunction occurs with injuries at or above T6, so monitoring of BP and heart rate will not be useful in determining the effectiveness of the medication. Respiratory and GI function will not be impaired by a T10 injury, so assessments of these systems will not provide information about whether the medication is effective.

In which order will the nurse perform the following actions when caring for a patient with possible cervical spinal cord trauma who is admitted to the emergency department? a. Administer O2 using a non-rebreathing mask. b. Monitor cardiac rhythm and blood pressure. c. Immobilize the patient's head, neck, and spine. d. Transfer the patient to radiology for spinal CT.

Correct Answer(s): C, A, B, D Rationale: The first action should be to prevent further injury by stabilizing the patient's spinal cord. Maintenance of oxygenation by administration of 100% O2 is the second priority. Because neurogenic shock is a possible complication, continuous monitoring of heart rhythm and BP is indicated. CT scan to determine the extent and level of injury is needed once initial assessment and stabilization is accomplished.

The nurse is caring for a patient admitted 1 week ago with an acute spinal cord injury. Which of the following assessment findings would alert the nurse to the presence of autonomic dysreflexia? A) Tachycardia B) Hypotension C) Hot, dry skin D) Throbbing headache

Correct Answer(s): D Autonomic dysreflexia is related to reflex stimulation of the sympathetic nervous system reflected by hypertension, bradycardia, throbbing headache, and diaphoresis.

When planning care for a patient with a C5 spinal cord injury, which nursing diagnosis is the highest priority? A) Risk for impairment of tissue integrity caused by paralysis B) Altered patterns of urinary elimination caused by quadriplegia C) Altered family and individual coping caused by the extent of trauma D) Ineffective airway clearance caused by high cervical spinal cord injury

Correct Answer(s): D Maintaining a patent airway is the most important goal for a patient with a high cervical fracture. Although all of these are appropriate nursing diagnoses for a patient with a spinal cord injury, respiratory needs are always the highest priority. Remember the ABCs.

A 70 yr old patient who has a spinal cord injury at C8 resulting in central cord syndrome. Which effect of the patient's most likely to be life threatening after completeing rehabiliation? A. increased bone density loss B. higher risk for tissue hypoxia C. vasomotor compensation lost D. Weakness of thoracic muscles

Correct Answer(s): D Weakness of thoracic muscle is most likely to cause life-threatening complications because affects patients oxygentation and ventilation.

Which statement by a high school athlete being discharged after experiencing a concussion indicates a need for more teaching? 1. "I can go to football practice tomorrow." 2. "I need to report a worsening headache to the provider." 3. "I'll have to be awakened every few hours when asleep." 4. "I can expect to be more fatigued for a while."

Correct Answer: 1 (1) Any patient with a concussion should rest for 48 hours after the injury, and teenagers may take longer. The student may not return to practice or play until a physician's release is obtained. (2, 3, 4) Are commonly recommended practices and indications.

A patient known to be positive for HIV is admitted with oral thrush, recurrent vaginal yeast infections, and skin infections. What do these signs indicate? 1. Opportunistic infection 2. Antimicrobial resistance 3. Resistant strain of HIV 4. Sentinel infection

Correct Answer: 1 Candidiasis is a common opportunistic infection (OI) in the HIV-positive patient. (2) Antimicrobial resistance can be determined only via microbiologic culture accompanied by the antibiogram (test that shows drug sensitivities). (3) Resistant strains of HIV are mutations of the virus that do not respond well to chemotherapy. (4) Sentinel infections are seen in AIDS-defining infections and where the candidiasis is currently located is not indicative of this type of infection

A nurse is reviewing medication orders for a female patient with SLE who is positive for the presence of antiphospholipid antibodies. The nurse would seek clarification from the provider about which type of medication? 1. Oral contraceptives 2. Hydroxychloroquine (antimalarial) 3. Glucocorticoid medication 4. NSAID

Correct Answer: 1 Women with lupus may need to have birth control with lower levels of estrogen to decrease flare-ups of systemic lupus erythematosus (SLE). (2) Hydroxychloroquine helps prevent organ damage in SLE. (3) Glucocorticoids help suppress the immune system, thereby lessening risk of flare-ups of SLE. (4) NSAIDs help in controlling paid associated with SLE.

A nurse keeps a postcraniotomy patient's neck in midline position and ensures that there is no excessive hip flexion. The rationale for the nurse's action would be that this position: 1. restores neutral position of the joints. 2. prevents a further increase in intracranial pressure. 3. promotes comfort and rest. 4. prevents the formation of blood clots.

Correct Answer: 2 (2) Keeping the neck in midline ensures proper drainage of fluids from the head; preventing excessive hip flexion seems to prevent increased intracranial pressure. (1) A neutral position is not necessarily recommended. (3) The midline position may not be comfortable for the patient. (4) The midline position does not prevent formation of blood clots.

A patient is admitted with a cardiac dysrhythmia. The morning laboratory values show potassium as 6.1 mg/dL. What action is most important to take? 1. Encourage intake of extra fluid. 2. Notify the provider immediately. 3. Check the breakfast tray for sodium-containing foods before serving. 4. Check the patient's vital signs.

Correct Answer: 2 A potassium level of 6.1 is above normal limits. Particularly in a patient with a heart problem, a high level of potassium may cause a serious dysrhythmia and the level should be lowered as quickly as possible. Call the physician and report the high level. Extra fluid is not indicated in this instance. Potassium, not sodium, is the problem. Increasing sodium intake is not recommended for a patient with heart problems. Monitoring the patient's vital signs is wise. Check to see if the heart rhythm is irregular, but the most important thing is to report the value to the physician.

A patient who had TURP complains of increasing bladder spasms. Which is an appropriate initial nursing action? 1. Medicate with a B&O suppository. 2. Check the urinary catheter tubing for kinks and obstruction. 3. Teach relaxation exercises. 4. Encourage use of patient-controlled analgesia.

Correct Answer: 2 Before giving medication, the nurse (2) checks to see that the tubing is not kinked and the catheter is draining well, as obstruction can cause bladder spasm. (1) This is not an initial action. Abdominal distention may be a sign of catheter obstruction as well. (3) Relaxation is not effective to eliminate spasms. (4) The patient who has had a radical procedure may have a patient-controlled analgesia pump to control pain.

A patient with high levels of serum ammonia asks, "Why do I have to continue taking lactulose?" What is the best response? 1. "It destroys ammonia-producing bacteria in the intestines." 2. "It reduces intestinal absorption of ammonia." 3. "It corrects vitamin B1 deficiency." 4. "It is used in preparation for a diagnostic test."

Correct Answer: 2 Lactulose is used to induce diarrhea and prevent diffusion of ammonia out of the intestinal tract. Neomycin is occasionally given orally or by enema to decrease the colonic bacteria. Thiamine is given to correct vitamin B1 deficiency. Lactulose does induce diarrhea, but it would not be the first choice for bowel preparation.

The nurse is reviewing the medications that each of her patients will receive during the shift. Which patient is likely to receive levothyroxine? 1. Patient who has von Recklinghausen disease 2. Patient who has hypothyroidism 3. Patient who has hyponatremia 4. Patient who has Graves disease

Correct Answer: 2 Levothyroxine is given to patients with hypothyroidism. Von Recklinghausen disease is hyperparathyroidism. Treatment may include infusion of isotonic sodium chloride and diuretic agents, phosphate therapy, and administration of calcitonin to decrease the rate of skeletal calcium release, or surgical removal of a major portion of the parathyroids. The underlying cause of hyponatremia must be determined, but fluid restriction and diuretics could be ordered if fluid overload is present. Graves' disease is a type of hyperthyroidism that may be treated with antithyroid drugs or radioactive iodine.

A patient has cirrhosis of the liver and ascites. The nurse should question which order? 1. Bed rest with bathroom privileges 2. Discontinue furosemide (Lasix) 80 mg 3. Give 2-g sodium diet 4. Fluid restriction 1500 mL/24 hr

Correct Answer: 2 Medical treatment includes administration of diuretics, bed rest, sodium, and fluid restriction.

There are two types of tumors: benign and malignant. A difference between the two types is: 1. malignant cells have a nucleus that is small and regular in shape, whereas benign cells are large and irregular. 2. malignant cells do not know when to stop multiplying, whereas benign cells have controlled patterns of reproduction and follow signals to stop. 3. malignant cells do not invade adjacent tissue, whereas benign cells sometimes do. 4. malignant cells reproduce exact copies, whereas benign cells become more disorganized with each succeeding generation.

Correct Answer: 2 One of the characteristics of malignant cells is that they become more and more undifferentiated as the malignancy progresses. The nucleus becomes large and irregular and they lose the trait of stopping reproduction upon command. (1) Malignant cells have a large nucleus. (3) Malignant cells can invade adjacent cells. (4) Benign cells are organized and malignant cells disorganized.

A 75-year-old patient who fell and hit his head a week ago is admitted for apparent personality changes, decreased level of consciousness, and irritability. The provider suspects a possible subdural hematoma. A family member asks about the condition. An accurate explanation would be: 1. "It is the presence of bleeding in the brain parenchyma." 2. "Bleeding occurs between the skull and the dura mater." 3. "It is the collection of blood between the brain and the inner surface of the dura mater." 4. "It is the intermittent blockage of circulation in various areas of the brain."

Correct Answer: 3 (3) A subdural hematoma occurs beneath the dura, between the brain and the dura. (1) The parenchyma is not between the brain and the inner surface of the dura mater. (2) The bleeding is on the other side of the dura mater. (4). A hematoma is not necessarily intermittent and occurs in a specific location.

The surgeon inserts an intraventricular catheter into the lateral ventricle of a patient with increased ICP. When asked by a relative about the procedure, an accurate response by the nurse would be: 1. "The catheter allows direct visualization of the brain tissue." 2. "The catheter is used to monitor brain waves." 3. "The catheter is used to remove excess fluid inside the brain." 4. "The catheter is used to infuse fluids and medications into the brain."

Correct Answer: 3 (3) An intraventricular catheter is used to drain off excess cerebral spinal fluid. (1) The catheter does not allow visualization of the brain tissue. (2) The catheter does not have a mechanism to monitor brain waves. (4). Fluids and medications are not infused into the brain through the intraventricular catheter.

The patient asks, "Why am I taking lisinopril (Zestril)?" An accurate statement by the nurse would be: 1. "The medication increases the force of contraction of the heart." 2. "The medication increases the heart rate." 3. "The medication helps prevent vasoconstriction." 4. "The medication causes excretion of extra fluid."

Correct Answer: 3 Lisinopril is an angiotensin-enzyme inhibitor and works to lower blood pressure and afterload by preventing vasoconstriction. This drug does not increase contractility of the heart. Lisinopril does not increase the heart rate. Lisinopril does not have a diuretic effect.

While discussing HF with a student, a nurse explains that the underlying weakness of the left ventricle results in reduced cardiac output and backup of fluid in the pulmonary system. The student nurse anticipates which sign/symptom? 1. Edema in the sacrum, legs, feet, and ankles 2. Hepatomegaly 3. Crackles in the lungs 4. Ascites

Correct Answer: 3 This patient has systolic heart failure because of difficulty in ejecting blood from the ventricle, and the signs and symptoms include crackles in the lungs. Peripheral edema will eventually occur but is not an acute sign of systolic failure. Hepatomegaly and ascites are late signs of heart failure.

Risperidone (Risperdal) is prescribed for an outpatient with moderate Alzheimer's disease (AD). Which information obtained by the nurse at the next clinic appointment indicates that the medication is effective? a. The patient has less agitation. b. The patient is dressed appropriately. c. The patient is able to swallow a pill. d. The patient's speech is clearer.

Correct Answer: A Rationale: Risperidone is an antipsychotic used to treat the agitation, aggression, and behavioral problems associated with AD. The other improvements might occur with cholinesterase inhibitors.

When developing a plan of care for a hospitalized patient with moderate dementia, which intervention will the nurse include? a. Reminding the patient frequently about being in the hospital b. Placing suction at the bedside to decrease the risk for aspiration c. Providing complete personal hygiene care for the patient d. Repositioning the patient frequently to avoid skin breakdown

Correct Answer: A Rationale: The patient with moderate dementia will have problems with short- and long-term memory and will need reminding about the hospitalization. The other interventions would be used for a patient with severe dementia, who would have difficulty with swallowing, self-care, and immobility.

Coexisting dementia and depression are identified in a patient with Parkinson's disease. The nurse anticipates that the greatest improvement in the patient's condition will occur with administration of a. antipsychotic drugs. b. anticholinergic agents. c. dopaminergic agents and antidepressant drugs. d. selective serotonin reuptake inhibitor (SSRI) agents.

Correct Answer: C Rationale: Parkinson's disease and depression are both potentially reversible conditions, and the patient's symptoms that are caused by these two conditions will improve with appropriate treatment. Anticholinergic agents are likely to worsen the patient's condition because they will block the effect of acetylcholine at the synaptic cleft. There is no indication that the patient needs an antipsychotic agent at this time. A selective serotonin reuptake inhibitor (SSRI) may be effective for the depression, but it does not address the patient's other conditions.

A 72-year-old patient hospitalized with pneumonia is disoriented and confused 2 days after admission. Which assessment information obtained by the nurse about the patient indicates that the patient is experiencing delirium rather than dementia? a. The patient is disoriented to place and time but oriented to person. b. The patient has a history of increasing confusion over several years. c. The patient's speech is fragmented and incoherent. d. The patient was oriented and alert when admitted.

Correct Answer: D Rationale: The onset of delirium occurs acutely. The degree of disorientation does not differentiate between delirium and dementia. Increasing confusion for several years is consistent with dementia. Fragmented and incoherent speech may occur with either delirium or dementia.

Which instructions should be given to a patient regarding preventing the spread of hepatitis A? (Select all that apply.) 1. Bleach solutions must be used to clean the bathroom. 2. Somebody else should be doing the cooking right now. 3. No vaccination is available for hepatitis A. 4. Good hand hygiene prevents the likelihood of passing the virus.

Correct Answers: 1, 2, 4 Hepatitis A vaccination is available and recommended. The other statements indicate an understanding of the information (see the Home Care Considerations box, Preventing the Spread of Hepatitis Virus, for additional information).

A nurse provides patient instructions regarding taking iodine preparations. It is important for the nurse to include which instruction(s)? (Select all that apply.) 1. "Dilute the preparations well." 2. "Use a straw to prevent staining of the teeth." 3. "Watch for easy bruising." 4. "Report severe epigastric pain." 5. "Anticipate a metallic taste."

Correct Answers: 1, 2, 4, 5 Iodine preparations should be given well-diluted and administered through a straw, as they can stain the teeth. Adverse effects of iodine preparations can include gastrointestinal upset, metallic taste, skin rashes, allergic reactions, and epigastric pain. Iodine does not cause bruising.

A nurse is sending a patient to the dialysis clinic. What predialysis nursing intervention should be included? (Select all that apply.) 1. Withholding anticoagulants 2. Administering antihypertensive 3. Assessing dialysis access site 4. Checking vital signs 5. Monitoring laboratory values

Correct Answers: 1, 3, 4, 5 (2) Antihypertensive medications are held because they can cause hypotension during the treatment. Nitroglycerin (NTG) patches, digitalis, and anticoagulants also are held. (1, 3-5) Physical assessment, checking for bruit and thrill at the access site, vital signs, weight and laboratory results are done and compared with post-treatment results.

What is included in the nursing care of a patient undergoing peritoneal dialysis? (Select all that apply.) 1. Maintain aseptic technique when accessing a peritoneal catheter. 2. Instruct the patient to remain supine until the dialysate is drained. 3. Weigh the patient before and after dialysis. 4. Monitor vital signs. 5. Check color and volume of effluent.

Correct Answers: 1, 3, 4, 5 (2) Patient can get up and ambulate after dialysate is instilled. (Dwell time varies from 4 to 8 hours. Others may have the dwell time at night or have continuous ambulatory peritoneal dialysis [CAPD], which goes on for 24 hours a day. (1, 3, 4, 5) All other answers are correct.

A nurse is caring for a patient after a thyroidectomy. What should the nurse monitor for? (Select all that apply.) 1. Bleeding and swelling 2. Hypothermia 3. Increase in pulse 4. Difficulty swallowing 5. Difficulty breathing

Correct Answers: 1, 3, 4, 5 Watch for signs of bleeding and swelling at the operative area. Any rise in temperature, pulse, or respiration rate should be reported immediately, as it may indicate a high level of thyroxine in the bloodstream. External swelling may cause constriction of the bandage around the neck. Difficulty in swallowing or breathing also should be reported immediately, as it may indicate internal edema and pressure on the esophagus and trachea. There would be no reason for the patient to be hypothermic.

A nurse answers the call light of a patient admitted with HF. The patient states he is short of breath and appears to be in distress. Identify the actions of the nurse in priority order. 1. Apply supplemental oxygen. 2. Raise the head of the bed. 3. Notify the provider. 4. Check vital signs. 5. Listen to lung sounds.

Correct Answers: 2, 1, 4, 5, 3 Raising the head of the bed can help immediately in easing breathing. Oxygen should be applied to supplement oxygen until further assessment can be done. Vital signs with pulse oximetry and assessment of lung sounds all need to be rechecked due to the patient's change in condition and to have the most current information available for the provider.

A health care provider who has not been immunized for hepatitis B is exposed to the hepatitis B virus (HBV) through a needle stick from an infected patient. The infection control nurse informs the individual that treatment for the exposure should include: a. baseline hepatitis B antibody testing now and in 2 months. b. active immunization with hepatitis B vaccine. c. hepatitis B immune globulin (HBIG) injection. d. both the hepatitis B vaccine and HBIG injection.

Correct D The recommended treatment for exposure to hepatitis B in unvaccinated individuals is to receive both HBIG and the hepatitis B vaccine, which would provide temporary passive immunity and promote active immunity. Antibody testing may also be done, but this would not provide protection from the exposure.

Which type of hepatitis is transmitted by the fecal-oral route via contaminated food, water, or direct contact with an infected person? 1. Hepatitis A 2. Hepatitis B 3. Hepatitis C 4. Hepatitis D

Correct answer: 1 Rationale: 1. The hepatitis A virus is in the stool of infected people for up to 2 weeks before symptoms develop 2. Hepatitis B is spread through contact with infected blood and body fluids 3. Hepatitis C is transmitted through contact with infected blood and body fluids 4. Hepatitis D infection only causes infection in people who are also infected with Hepatitis B or C

The nurse is caring for a patient with amyotrophic lateral sclerosis (ALS). The nurse plans care based on which understanding of the patient's prognosis? 1. The disease progresses slowly and is fatal. 2. The disease will progress over many years but the patient's quality of life will be good. 3. The disease progresses rapidly but can be halted by drug therapy. 4. The disease will progress slowly and can be controlled by medication.

Correct answer: 1 Rationale 1: The disease is slowly progressive and fatal and is characterized by weakness and wasting of muscles under voluntary control. Rationale 2: Quality of life is profoundly affected by this disorder. Rationale 3: Riluzole (Rilutek) is available to treat the disease, but it does not halt it. Death usually occurs due to respiratory failure. Rationale 4: The disease is slowly progressive and cannot be controlled by medication.

A diagnosis of AIDS is made when an HIV-infected patient has a. a CD4+ T cell count below 200/µL. b. a high level of HIV in the blood and saliva. c. lipodystrophy with metabolic abnormalities. d. oral hairy leukoplakia, an infection caused by Epstein-Barr virus.

Correct answer: A Rationale: AIDS is diagnosed when an individual with HIV infection meets one of several criteria; one criterion is a CD4+ T cell count below 200 cells/L. Other criteria are listed in Table 15-9.

During HIV infection a. the virus replicates mainly in B-cells before spreading to CD4+ T cells. b. infection of monocytes may occur, but antibodies quickly destroy these cells. c. the immune system is impaired predominantly by the eventual widespread destruction of CD4+ T cells. d. a long period of dormancy develops during which HIV cannot be found in the blood and there is little viral replication

Correct answer: C Rationale: Immune dysfunction in HIV disease is caused predominantly by damage to and destruction of CD4+ T cells (i.e., T helper cells or CD4+ T lymphocytes)

The patient with advanced cirrhosis asks why his abdomen is so swollen. The nurse's response is based on the knowledge that a. a lack of clotting factors promotes the collection of blood in the abdominal cavity. b. portal hypertension and hypoalbuminemia cause a fluid shift into the peritoneal space. c. decreased peristalsis in the GI tract contributes to gas formation and distention of the bowel. d. bile salts in the blood irritate the peritoneal membranes, causing edema and pocketing of fluid.

Correct answer: b Rationale: Ascites is the accumulation of serous fluid in the peritoneal or abdominal cavity and is a common manifestation of cirrhosis. With portal hypertension, proteins shift from the blood vessels through the larger pores of the sinusoids (capillaries) into the lymph space. When the lymphatic system is unable to carry off the excess proteins and water, those substances leak through the liver capsule into the peritoneal cavity. Osmotic pressure of the proteins pulls additional fluid into the peritoneal cavity. A second mechanism of ascites formation is hypoalbuminemia, which results from the inability of the liver to synthesize albumin. Hypoalbuminemia results in decreased colloidal oncotic pressure. A third mechanism is hyperaldosteronism, which occurs when aldosterone is not metabolized by damaged hepatocytes. The increased level of aldosterone causes increases in sodium reabsorption by the renal tubules. Sodium retention and an increase in antidiuretic hormone levels cause additional water retention.

Which statements accurately describe HIV infection (select all that apply)? a. Untreated HIV infection has a predictable pattern of progression. b. Late chronic HIV infection is called acquired immunodeficiency syndrome (AIDS). c. Untreated HIV infection can remain in the early chronic stage for a decade or more. d. Untreated HIV infection usually remains in the early chronic stage for 1 year or less. e. Opportunistic diseases occur more often when the CD4+ T cell count is high and the viral load is low

Correct answers: a, b, c Rationale: The typical course of untreated HIV infection follows a predictable pattern. However, treatment can significantly alter this pattern, and disease progression is highly individualized. Late chronic infection is another term for acquired immunodeficiency syndrome (AIDS). The median interval between untreated HIV infection and a diagnosis of AIDS is about 11 years.

Which statement about metabolic side effects of ART is true (select all that apply)? a. These are annoying symptoms that are ultimately harmless. b. ART-related body changes include central fat accumulation and peripheral wasting. c. Lipid abnormalities include increases in triglycerides and decreases in high-density cholesterol. d. Insulin resistance and hyperlipidemia can be treated with drugs to control glucose and cholesterol. e. Compared to uninfected people, insulin resistance and hyperlipidemia are more difficult

Correct answers: b, c, d Rationale: Some HIV-infected patients, especially those who have been infected and have received ART for a long time, develop a set of metabolic disorders that include changes in body shape (e.g., fat deposits in the abdomen, upper back, and breasts along with fat loss in the arms, legs, and face) as a result of lipodystrophy, hyperlipidemia (i.e., elevated triglyceride levels and decreases in high-density lipoprotein levels), insulin resistance and hyperglycemia, bone disease (e.g., osteoporosis, osteopenia, avascular necrosis), lactic acidosis, and cardiovascular disease.

A patient with hepatitis B is being discharged in 2 days. In the discharge teaching plan the nurse should include instructions to: A. Avoid alcohol for the first 3 weeks B.use condoms during sexual intercourse C. have family members get an injection of immunoglobulin D. follow low protein, moderate carb, moderate fat diet"

Correct: B B. is the correct answer as it is important to instruct the patient they this disease can be spread through sexual contact

You're patient who had a stroke has issues with understanding speech. What type of aphasia is this patient experiencing and what area of the brain is affected? A. Expressive; Wernicke's area B. Receptive, Broca's area C. Expressive; hippocampus D. Receptive; Wernicke's area

D

The decision to begin antiretroviral therapy is based on: A. The CD4 cell count B. The plasma viral load C. The intensity of the patient's clinical symptoms D. All of the above

D A person's CD4 count is an important factor in the decision to start ART. A low or falling CD4 count indicates that HIV is advancing and damaging the immune system. A rapidly decreasing CD4 count increases the urgency to start ART. Regardless of CD4 count, there is greater urgency to start ART when a person has a high viral load or any of the following conditions: pregnancy, AIDS, and certain HIV-related illnesses and co infections.

A client with ALS is having trouble swallowing. What is the next intervention to maintain the client's nutritional status? a. total parenteral nutrition b. liquid protein diet c. nasoenteric feeding tube d. gastrostomy tube

D A regular diet adapted to provide soft, easily chewed food is maintained as long as the client can swallow. A tube feeding is required to prevent aspiration as chewing and swallowing difficulties arise. A gastrostomy tube is preferred over a nasoenteric tube for long term feeding.

For a male client who has acquired immunodeficiency syndrome with chronic diarrhea, anorexia, a history of oral candidiasis, and weight loss, which dietary instruction would be included in the teaching plan? A. "Follow a low-protein, high-carbohydrate diet." B. "Eat three large meals per day." C. "Include unpasteurized dairy products in the diet." D. "Follow a high-protein, high-calorie diet.

D Dietary instructions should include the need for a high-protein, high-calorie diet. The patient should be taught to eat small, frequent meals and include low-microbial foods, such as pasteurized dairy products, washed and peeled fruits and vegetables, and well-cooked meats.

A client tells the nurse that he is experiencing some leg stiffness when walking and slowness when performing ADLs. Occasionally he has noted slight tremors in his hands at rest. This in-formation leads the nurse to suspect a. amyotrophic lateral sclerosis (ALS). b. Huntington's disease. c. myasthenia gravis (MG). d. Parkinson's disease (PD).

D Early in PD the client may notice a slight slowing in the ability to perform ADLs. A general feeling of stiffness may be noticed, along with mild, diffuse muscular pain. Tremor is a common early manifestation that usually occurs in one of the upper limbs.

The early manifestations of amyotrophic lateral sclerosis (ALS) and multiple sclerosis (MS) are somewhat similar. What clinical feature of ALS distinguishes it from MS? A. Dysarthria B. Dysphagia C. Muscle weakness D. Impairment of respiratory muscles

D In ALS, there is progressive muscle atrophy until a flaccid quadriplegia develops. Eventually, there is involvement of the respiratory muscles, which leads to respiratory compromise.

A client with acquired immunodeficiency syndrome (AIDS) has a nursing diagnosis of Imbalanced nutrition: less than body requirements. The nurse plans which of the following goals with this client? a) consume foods and beverages that are high in glucose b) plan large menus and cook meals in advance c) eat low-calorie snacks between meals d) eat small, frequent meals throughout the day

D The client should eat small, frequent meals throughout the day. The client also should take in nutrient-dense and high-calorie meals and snacks rather than those that are high in glucose only. The client is encouraged to eat favorite foods to keep intake up and plan meals that are easy to prepare. The client can also avoid taking fluids with meals to increase food intake before satiety sets in.

Which assessment finding represents a positive response to erythropoietin (Epogen, Procrit) therapy? A Hematocrit of 26.7% B Potassium within normal range C Absence of spontaneous fractures D Less fatigue

D Less fatigue Treatment of anemia with erythropoietin will result in increased hemoglobin and hematocrit (H&H) and decreased shortness of breath and fatigue. A hematocrit value of 26.7% is low; erythropoietin should restore the hematocrit to at least 36% to be effective. Erythropoietin causes more red blood cells to be produced, increasing H&H, not potassium. Calcium supplements and phosphate binders prevent renal osteodystrophy; erythropoietin treats anemia.

When caring for a client who receives peritoneal dialysis (PD), which finding does the nurse report to the provider immediately? A Pulse oximetry reading of 95% B Sinus bradycardia, rate of 58 beats/min C Blood pressure of 148/90 mm Hg D Temperature of 101.2° F (38.4° C)

D Temperature of 101.2° F (38.4° C) Peritonitis is the major complication of PD, caused by intra-abdominal catheter site contamination; meticulous aseptic technique must be used when caring for PD equipment. A pulse oximetry reading of 95% is a normal saturation. Although a heart rate of 58 beats/min is slightly bradycardic, the provider can be informed upon visiting the client. Clients with kidney failure tend to have slightly higher blood pressures due to fluid retention; this is not as serious as a fever.

The teaching plan for a client being started on long-acting nitroglycerin includes the action of this drug. The nurse teaches that this drug relieves chest pain by which action? A) Dilating just the coronary arteries B) Decreasing the blood pressure C) Increasing contractility of the heart D) Dilating arteries and veins

D) Dilating arteries and veins

Which of the following conditions is the predominant cause of angina? A) Increased preload B) Decreased afterload C) Coronary artery spasm D) Inadequate oxygen supply to the myocardium

D) Inadequate oxygen supply to the myocardium Inadequate oxygen supply to the myocardium is responsible for the pain accompanying angina. Increased preload would be responsible for right-sided heart failure. Decreased afterload causes increased cardiac output. Coronary artery spasm is responsible for variant angina.

What is the primary reason for administering morphine to a client with an MI? A) To sedate the client B) To decrease the client's pain C) To decrease the client's anxiety D) To decrease oxygen demand on the client's heart

D) To decrease oxygen demand on the client's heart Morphine is administered because it decreases myocardial oxygen demand. Morphine will also decrease pain and anxiety while causing sedation, but it isn't primarily given for those reasons

The nurse suspects that a client has developed an acute arterial occlusion of the right lower extremity based on which signs/symptoms? (Select all that apply.) A. Hypertension B. Tachycardia C. Bounding right pedal pulses D. Cold right foot E. Numbness and tingling of right foot F. Mottling of right foot and lower leg

D, E, F Pain, pallor, pulselessness, paresthesia, paralysis, poikilothermia (cool limb), and mottled color are characteristics of acute arterial occlusion.

When teaching about clopidogrel (Plavix), the nurse will tell the patient with cerebral atherosclerosis a. to monitor and record the blood pressure daily. b. that Plavix will dissolve clots in the cerebral arteries. c. that Plavix will reduce cerebral artery plaque formation. d. to call the health care provider if stools are bloody or tarry.

D. Clopidogrel (Plavix) inhibits platelet function and increases the risk for gastrointestinal bleeding, so patients should be advised to notify the health care provider about any signs of bleeding. The medication does not lower blood pressure, decrease plaque formation, or dissolve clots.

A nurse is completing the admission assessment of a client who has acute pancreatitis. Which finding is the first priority? A) History of cholelithiasis B) Elevated serum amylase levels C) Decrease in bowel sounds upon auscultation D) Hand spasms present when blood pressure is checked

D. Rationale: the greatest risk to the client is ECG changes and hypotension from hypocalcemia. Hand spasms when taking blood pressure is a manifestation of hypocalcemia

A 73-year-old patient with a stroke experiences facial drooping on the right side and right-sided arm and leg paralysis. When admitting the patient, which clinical manifestation will the nurse expect to find? a. Impulsive behavior b. Right-sided neglect c. Hyperactive left-sided tendon reflexes d. Difficulty comprehending instructions

D. Right-sided paralysis indicates a left-brain stroke, which will lead to difficulty with comprehension and use of language. The left-side reflexes are likely to be intact. Impulsive behavior and neglect are more likely with a right-side stroke.

Your patient has been diagnosed with MS. You are teaching her about how to reduce muscle spasticity. Which of the following statements, if made by the patient would indicate the need for further teaching? A) Daily exercise, including weight bearing can help relieve spasticity B) My stretching routine can help with the spasms C) Taking Baclofen may help relieve these painful spasms in my legs D) At the end of a day, taking a nice hot bath may relieve the muscle spasms

D. The patient with MS should never use hot water for a bath due to sensory deficits. All other answers can help with muscle spasms. Warm compresses can be used to relieve muscle spasms.

A 56-year-old patient arrives in the emergency department with hemiparesis and dysarthria that started 2 hours previously, and health records show a history of several transient ischemic attacks (TIAs). The nurse anticipates preparing the patient for a. surgical endarterectomy. b. transluminal angioplasty. c. intravenous heparin administration. d. tissue plasminogen activator (tPA) infusion.

D. The patient's history and clinical manifestations suggest an acute ischemic stroke and a patient who is seen within 4.5 hours of stroke onset is likely to receive tPA (after screening with a CT scan). Heparin administration in the emergency phase is not indicated. Emergent carotid transluminal angioplasty or endarterectomy is not indicated for the patient who is having an acute ischemic stroke.

Which statement by the patient indicates an understanding of discharge instructions given by the nurse about the newly prescribed medication levothyroxine (Synthroid)? A. "I will take a double dose to make up for the missed one." B. "I can expect improvement of my symptoms within 1 week." C. "I will stop the medication immediately if I feel pain or weakness in my muscles." D. "I will take this medication in the morning so it does not affect my sleep at night."

D. "I will take this medication in the morning so it does not affect my sleep at night." Levothyroxine increases basal metabolic rate and thus may cause insomnia. Patients should not double the dose or stop taking the medication abruptly. It may take up to 4 weeks for a therapeutic response to occur.

A patient who has been prescribed an antineoplastic drug for his newly diagnosed cancer asks the nurse what the most common side effect is for these drugs. What is the nurse's most accurate response? A. "An elevated temperature is a common side effect." B. "Gastrointestinal upset is the most common side effect." C. "These drugs almost always cause a vitamin B12 deficiency." D. "Most of these drugs cause some degree of bone-marrow depression."

D. "Most of these drugs cause some degree of bone-marrow depression." All antineoplastic drugs cause bone-marrow depression. The degree of bone-marrow depression depends on the drug and dosage. Gastrointestinal upset is experienced with some neoplastic drugs. An elevated temperature is a sign of infection, and vitamin B12 deficiency is not commonly a side effect.

A patient who has undergone transurethral resection of the prostate (TURP) surgery asks why he needs to have the continuous bladder irrigation (CBI) because it seems to increase his pain. Which explanation would be the best? A. "Normal urine production is maintained until healing can occur." B. "Antibiotics are being instilled into the bladder to prevent infection." C. "The bladder irrigation is necessary to stop the bleeding in the bladder." D. "The irrigation is needed to keep the catheter from becoming occluded by blood clots."

D. "The irrigation is needed to keep the catheter from becoming occluded by blood clots." The continuous irrigation acts to flush small to moderate clots from the GU tract, keeping it patent. CBIs are not used to maintain normal urine production, instill antibiotics into the bladder, or stop bleeding in the bladder.

A patient who has been prescribed an antineoplastic drug for his newly diagnosed cancer asks the nurse what the most common side effect is for these drugs. What is the nurse's most accurate response? A. "These drugs almost always cause a vitamin B12 deficiency." B. "Gastrointestinal (GI) upset is the most common side effect." C. "An elevated temperature is a common side effect." D. "Most of these drugs cause some degree of bone marrow depression."

D. All antineoplastic drugs cause bone marrow depression. The degree of bone marrow depression depends on the drug and dosage. GI upset is experienced with some neoplastic drugs. An elevated temperature is a sign of infection, and vitamin B

The health care provider orders lactulose for a patient with hepatic encephalopathy. The nurse will monitor for effectiveness of this medication for this patient by assessing what? A. Relief of constipation B. Relief of abdominal pain C. Decreased liver enzymes D. Decreased ammonia levels

D. Decreased ammonia levels Hepatic encephalopathy is a complication of liver disease and is associated with elevated serum ammonia levels. Lactulose traps ammonia in the intestinal tract. Its laxative effect then expels the ammonia from the colon, resulting in decreased serum ammonia levels and correction of hepatic encephalopathy.

In the postoperative period, the LPN/LVN should observe a patient who has had a thyroidectomy for which signs of thyroid crisis? A. Depression and fatigue B. Respiratory distress and hoarseness C. Twitching of muscles and severe convulsions D. Extreme temperature elevation and rapid pulse rate

D. Extreme temperature elevation and rapid pulse rate Thyroid storm (TS), also known as thyroid crisis or thyrotoxicosis, is another complication following a thyroidectomy. In the postoperative setting, the condition is caused by a sudden increase in the output of thyroxine caused by manipulation of the thyroid as it is being removed. Another cause of TS may be improper reduction of thyroid secretions before surgery. The symptoms of TS are produced by a sudden and extreme elevation of all body processes. The temperature may rise to 106° F (41.1° C) or more, the pulse increases to as much as 200 beats/min, respirations become rapid, and the patient exhibits marked apprehension and restlessness. Unless the condition is relieved, the patient quickly passes from delirium to coma to death from heart failure. Respiratory distress is a complication of thyroidectomy if the edema affects the airway, but it is not a sign of thyroid storm. Muscle twitching and convulsions are a sign of hypocalcemia from hyperparathyroidism. Hoarseness is an expected finding following thyroidectomy. Depression and fatigue may result from hypothyroidism.

The patient is being discharged following a mitral valve replacement and will be prescribed the anticoagulant warfarin. Which food should the nurse instruct him to avoid? A. Bananas B. Dairy products C. All meat products D. Green leafy vegetables

D. Green leafy vegetables Green leafy vegetables contain a significant amount of vitamin K, which would counteract the patient's anticoagulant (warfarin). Bananas, dairy products, and meat products do not need to be avoided.

A patient reports they do not eat enough iodine in their diet. What condition are they most susceptible to? A. Pheochromocytoma B. Hyperthyroidism C. Thyroid Storm D. Hypothyroidism

D. Hypothyroidism Iodine helps make T3 and T4....if a person does not consume enough iodine they are at risk for developing HYPOTHYROIDISM.

When planning care for a patient with cirrhosis, the nurse will give highest priority to which nursing diagnosis? A. Impaired skin integrity related to edema, ascites, and pruritus B. Imbalanced nutrition: less than body requirements related to anorexia C. Excess fluid volume related to portal hypertension and hyperaldosteronism D. Ineffective breathing pattern related to pressure on diaphragm and reduced lung volume

D. Ineffective breathing pattern related to pressure on diaphragm and reduced lung volume Although all of these nursing diagnoses are appropriate and important in the care of a patient with cirrhosis, airway and breathing are always the highest priorities.

The nurse is teaching a patient taking an antithyroid medication to avoid food items high in iodine. Which food item should the nurse instruct the patient to avoid? A. Milk B. Eggs C. Chicken D. Seafood

D. Seafood Seafood contains high amounts of iodine. The other choices do not.

A patient is 6 hours post-opt from a thyroidectomy. The surgical site is clean, dry and intact with no excessive swelling noted. What position is best for this patient to be in? A. Fowler's B. Prone C. Trendelenburg D. Semi-Fowler's

D. Semi-Fowler's

The client is diagnosed with acute pancreatitis. What health-care provider's admitting order should the nurse question? A. Bedrest with bathroom privileges B. Initiate IV therapy of D5W at 125 mL/hr C. Weight client daily D. Low fat, low carb diet

D. carbs are the easiest for the body to digest Rationale: Bedrest decreases the metabolic rate. The client should be NPO to rest the pancreas to decrease the auto digestion of the pancreas. Since the client is NPO IV therapy is appropriate. Weight changes will happen as a result of diet and IV fluids therefore daily weights is appropriate.

Which condition is NOT a known cause of cirrhosis? A. Obesity B. Alcohol consumption C. Blockage of the bile duct D. Hepatitis C E. All are known causes of cirrhosis

E. All are known causes of cirrhosis

True or false, in clients with closed head injury, a lumbar puncture is necessary to assess for presence of blood in the cerebral spinal fluid (CSF)?

False

A patient with late-stage cirrhosis develops portal hypertension. Which of the following options below are complications that can develop from this condition? Select all that apply: A. Increase albumin levels B. Ascites C. Splenomegaly D. Fluid volume deficient E. Esophageal varices

The answer are B, C, and E. Portal Hypertension is where the portal vein becomes narrow due to scar tissue in the liver, which is restricting the flow of blood to the liver. Therefore, pressure becomes increased in the portal vein and affects the organs connected via the vein to the liver. The patient may experience ascites, enlarged spleen "splenomegaly", and esophageal varices etc.

You're providing care to a patient with a uric acid kidney stone that is 2 mm in size per diagnostic imaging. The patient is having severe pain and rates their pain 10 on 1-10 scale. The physician has ordered a treatment plan to assist the patient in passing the kidney stone. What nursing intervention is PRIORITY for this patient based on the scenario? A. Administer pain medication B. Encourage fluid intake of 2-4 liters per day C. Massage the costovertebral area D. Implement a high protein diet

The answer is A. Controlling the patient's pain is priority. Option B is another important part of the patient's plan of care to help assist the passage of the kidney stone, but it is not priority at the moment until the patient's pain is controlled. Option C and D are not recommended for the treatment of uric acid kidney stones. You would never massage the costovertebral area, and a high protein diet will further increase uric acid levels, therefore, should be avoided.

Which patient below with acute kidney injury is in the oliguric stage of AKI: A. A 56 year old male who has metabolic acidosis, decreased GFR, increased BUN/Creatinine, hyperkalemia, edema, and urinary output 350 mL/day. B. A 45 year old female with metabolic alkalosis, hypokalemia, normal GFR, increased BUN/creatinine, edema, and urinary output 600 mL/day. C. A 39 year old male with metabolic acidosis, hyperkalemia, improving GFR, resolving edema, and urinary output 4 L/day. D. A 78 year old female with respiratory acidosis, increased GFR, decreased BUN/creatinine, hypokalemia, and urinary output 550 mL/day.

The answer is A. During the oliguric stage of AKI the patient will have a urinary output of 400 mL/day or LESS. This is due to a decreased GRF (glomerular filtration rate), which will lead to increased amounts of waste in the blood (increased BUN/Creatinine), metabolic acidosis (decreased excretion of hydrogen ions), hyperkalemia, hypervolemia (edema/hypertension), and urinary output of <400 mL/day.

A patient is admitted with hepatic encephalopathy secondary to cirrhosis. Which meal option selection below should be avoided with this patient? A. Beef tips and broccoli rabe B. Pasta noodles and bread C. Cucumber sandwich with a side of grapes D. Fresh salad with chopped water chestnuts

The answer is A. Patients who are experiencing hepatic encephalopathy are having issues with toxin build up in the body, specifically ammonia. Remember that ammonia is the byproduct of protein breakdown, and normally the liver can take the ammonia from the protein breakdown and turn it into urea (but if the cirrhosis is severe enough this can't happen). Therefore, the patient should consume foods LOW in protein until the encephalopathy subsides. Option A is very high in protein while the others are low in protein. Remember meats, legumes, eggs, broccoli rabe, certain grains etc. are high in protein.

While assisting a patient with chronic pancreatitis to the bathroom, you note the patient's stool to be oily/greasy in appearance. In your documentation you note this as: A. Steatorrhea B. Melena C. Currant D. Hematochezia

The answer is A. Steatorrhea is an oily/greasy appearance of the stool which can occur in chronic pancreatitis. This occurs due to the inability of the pancreas to produce digestive enzymes which help break down fats. Fats are not being broken down; therefore, it is being excreted into the stool. Melena is used to describe tarry/black stool, hematochezia is used to describe red stools, and currant are jelly type stools.

A patient with stage 4 chronic kidney disease asks what type of diet they should follow. You explain the patient should follow a: A. Low protein, low sodium, low potassium, low phosphate diet B. High protein, low sodium, low potassium, high phosphate diet C. Low protein, high sodium, high potassium, high phosphate diet D. Low protein, low sodium, low potassium, high phosphate diet

The answer is A. The patient should follow this type of diet because protein breaks down into urea (remember patients will have increased urea levels), low sodium to prevent fluid retention, low potassium to prevent hyperkalemia (remember glomerulus isn't filtering out potassium/phosphate as it should), and low phosphate to prevent hyperphosphatemia.

A 65 year old male patient has a glomerular filtration rate of 55 mL/min. The patient has a history of uncontrolled hypertension and coronary artery disease. You're assessing the new medication orders received for this patient. Which medication ordered by the physician will help treat the patient's hypertension along with providing a protective mechanism to the kidneys? A. Lisinopril B. Metoprolol C. Amlodipine D. Verapamil

The answer is A. There are two types of drugs that can be used to treat hypertension and protect the kidneys in patients with CKD. These drugs include angiotensin converting enzyme inhibitors (ACE inhibitors) and angiotensin receptor blockers (ARBs). The only drug listed here that is correct is Lisinopril. This drug is known as an ACE inhibitor. Metoprolol is a BETA BLOCKER. Amlodipine and Verapamil are calcium channel blockers.

You're assessing morning lab values on a female patient who is recovering from a myocardial infraction. Which lab value below requires you to notify the physician? A. Potassium level 4.2 mEq/L B. Creatinine clearance 35 mL/min C. BUN 20 mg/dL D. Blood pH 7.40

The answer is B. A normal creatinine clearance level in a female should be 85-125 mL/min (95-140 mL/min males). A creatinine clearance level indicates the amount of blood the kidneys can make per minute that contain no amounts of creatinine in it. Remember creatinine is a waste product of muscle breakdown. Therefore, the kidneys should be able to remove excessive amounts of it from the bloodstream. A patient who has experienced a myocardial infraction is at risk for pre-renal acute injury due to decreased cardiac output to the kidneys from a damaged heart muscle (the heart isn't able to pump as efficiently because of ischemia). All the other labs values are normal.

You are providing pre-op teaching to a patient scheduled for a percutaneous nephrolithotomy. Which statement by the patient demonstrates the patient understood the pre-op teaching? A. "During the procedure the surgeon will move the stone down the ureter, so I can pass the stone in the urine. B. "I may have a nephrostomy tube after the procedure." C. "A scope is inserted through the urinary system from the urethra to the kidneys to assess the kidney stone." D. "This procedure is noninvasive and no incision is required."

The answer is B. A percutaneous nephrolithotomy is an INVASIVE procedure that can be used to remove large kidney stones. An incision is made on the back where the kidney is located. A nephroscope is then insert through the incision and used to remove the stone. Generally, the surgeon is able to remove the stone or break it up, therefore, the patient doesn't have to pass it naturally as with other procedures. A nephrostomy tube is sometimes placed after the procedure to drain urine and fragments of the stone out of the kidney.

A patient with CKD has a low erythropoietin (EPO) level. The patient is at risk for? A. Hypercalcemia B. Anemia C. Blood clots D. Hyperkalemia

The answer is B. EPO (erythropoietin) helps create red blood cells in the bone marrow. The kidneys produce EPO and when the kidneys are damaged in CKD they can decrease in the production of EPO. Therefore, the patient is at risk for anemia.

A patient is admitted with uncontrolled atrial fibrillation. The patient's medication history includes vitamin D supplements and calcium. What type of stroke is this patient at MOST risk for? A. Ischemic thrombosis B. Ischemic embolism C. Hemorrhagic D. Ischemic stenosis

The answer is B. If a patient is in uncontrolled a-fib they are at risk for clot formation within the heart chambers. This clot can leave the heart and travel to the brain. Hence, an ischemic embolism type stroke can occur. An ischemic thrombosis type stroke is where a clot forms within the artery wall of the neck or brain.

You're developing a nursing care plan for a patient with a kidney stone. Which of the following nursing interventions will you include in the patient's plan of care? A. Restrict calcium intake B. Strain urine with every void C. Keep patient in supine position to alleviate pain D. Maintain fluid restriction of 1-2 Liter per day

The answer is B. It is vital the nurse strains every void and assesses the urine very closely for stones. This is crucial so it can be determined what type of kidney stone is causing the problem, therefore, appropriate treatment can be ordered. Restricting calcium intake is no longer recommended unless the patient has a metabolic or renal tubule problem. It is important to avoid placing the patient in the supine position for long periods because this impedes the flow of urine and the patient's ability to pass the stone. Fluid should not be restricted (unless the patient has a condition that requires it like heart failure etc.) because this concentrates the urine...hence increases the chances of another stone developing.

A patient with acute pancreatitis is reporting excessive thirst, excessive voiding, and blurred vision. As the nurse, it is priority you? A. Reassure the patient this is normal with pancreatitis B. Check the patient's blood glucose C. Assist the patient with drinking a simple sugar drink like orange juice D. Provide a dark and calm environment

The answer is B. Patients with acute pancreatitis are at risk for hyperglycemia (the signs and symptoms the patient are reporting are classic symptoms of hyperglycemia). Remember the endocrine function of the pancreas (which is to release insulin/glucagon etc. is insufficient) so the nurse must monitor the patient's blood glucose levels even if the patient is not diabetic.

A patient's MRI imaging shows damage to the cerebellum a week after the patient suffered a stroke. What assessment findings would correlate with this MRI finding? A. Vision problems B. Balance impairment C. Language difficulty D. Impaired short-term memory

The answer is B. The cerebellum is important for coordination and balance.

You're providing an in-service to a group of nurses about the different types of kidney stones. You explain to the attendees that the most common type of kidney stone is made up of: A. Cholesterol B. Calcium and oxalate C. Calcium and phosphate D. Uric acid

The answer is B. The most common type of kidney stone composite is calcium and oxalate.

A patient with Stage 5 CKD is experiencing extreme pruritus and has several areas of crystallized white deposits on the skin. As the nurse, you know this is due to excessive amounts of what substance found in the blood? A. Calcium B. Urea C. Phosphate D. Erythropoietin

The answer is B. This patient is experiencing uremic frost that occurs in severe chronic kidney disease. This is due to high amounts of urea in the blood being secreted via the sweat glands onto the skin, which will appear as white deposits on the skin. The patient will experience itching with this as well.

Which patient below is at most risk for a hemorrhagic stroke? A. A 65 year old male patient with carotid stenosis. B. A 89 year old female with atherosclerosis. C. A 88 year old male with uncontrolled hypertension and a history of brain aneurysm repair 2 years ago. D. A 55 year old female with atrial flutter.

The answer is C. A hemorrhagic stroke occurs when bleeding in the brain happens due to a break in a blood vessel. Risk factors for a hemorrhagic stroke is uncontrolled hypertension, history of brain aneurysm, old age (due to aging blood vessels.) All the other options are at risk for an ischemic type of stroke.

During your morning assessment of a patient with cirrhosis, you note the patient is disoriented to person and place. In addition while assessing the upper extremities, the patient's hands demonstrate a flapping motion. What lab result would explain these abnormal assessment findings? A. Decreased magnesium level B. Increased calcium level C. Increased ammonia level D. Increased creatinine level

The answer is C. Based on the assessment findings and the fact the patient has cirrhosis, the patient is experiencing hepatic encephalopathy. This is due to the buildup of toxins in the blood, specifically ammonia. The flapping motion of the hands is called "asterixis". Therefore, an increased ammonia level would confirm these abnormal assessment findings.

Which patient below is at MOST risk for developing uric acid type kidney stones? A. A 53 year old female with recurrent urinary tract infections. B. A 6 year old male with cystinuria. C. A 63 year male with gout. D. A 25 year old female that follows a vegan diet and report eating high amounts of spinach and strawberries on a regular basis.

The answer is C. Patients with gout experience high uric acid levels which can lead to the development of uric acid kidney stones. In option A, the patient is at risk for struvite kidney stones. In option B, the patient is at risk for cystine kidney stones, and in option C, the patient is at a small risk for calcium oxalate stones due to the high consumption of foods with oxalates.

A 55 year old male patient is admitted with a massive GI bleed. The patient is at risk for what type of acute kidney injury? A. Post-renal B. Intra-renal C. Pre-renal D. Intrinsic renal

The answer is C. Pre-renal injury is due to decreased perfusion to the kidneys secondary to a cause (massive GI bleeding...patient is losing blood volume). This leads to a major decrease in kidney function because the kidneys are deprived of nutrients to function and the amount of blood it can filter. Pre-renal injury can eventually lead to intrarenal damage where the nephrons become damaged.

A patient with AKI has a urinary output of 350 mL/day. In addition, morning labs showed an increased BUN and creatinine level along with potassium level of 6 mEq/L. What type of diet ordered by the physician is most appropriate for this patient? A. Low-sodium, high-protein, and low-potassium B. High-protein, low-potassium, and low-sodium C. Low-protein, low-potassium, and low-sodium D. High-protein and high-potassium

The answer is C. The patient with AKI, especially in the oliguric stage of AKI, should eat a low-protein, low-potassium, and low-sodium diet. This is because the kidneys are unable to filter out waste products, excessive water, and maintain electrolyte balance. The patient will have a buildup of waste (BUN and creatinine). Remember these waste products are the byproduct of protein (urea) and muscle breakdown (creatinine). So the patient should avoid high-protein foods. In addition, the patient is at risk for hyperkalemia and fluid overload (needs low-potassium and sodium foods).

In order for tissue plasminogen activator (tPA) to be most effective in the treatment of stroke, it must be administered? A. 6 hours after the onset of stroke symptoms B. 3 hours before the onset of stroke symptoms C. 3 hours after the onset of stroke symptoms D. 12 hours before the onset of stroke symptoms

The answer is C. tPa dissolves the clot causing the blockage in stroke by activating the protein that causes fibrinolysis. It should be given within 3 hours after the onset of stroke symptoms. It can be given 3 to 4.5 hours after onset IF the patient meets strict criteria. It is used for acute ischemia stroke, NOT hemorrhagic!!

Which of the following is NOT a role of the liver? A. Removing hormones from the body B. Producing bile C. Absorbing water D. Producing albumin

The answer is C. The liver does not absorb water. The intestines are responsible for this function.

The physician orders a patient with pancreatitis to take a pancreatic enzyme. What assessment finding demonstrates the pancreatic enzymes are working properly? A. Abdominal girth is decreased B. Skin turgor is less than 2 seconds C. Blood glucose is 250 D. Stools appear formed and solid

The answer is D. Pancreatic enzymes help the body break down carbs, proteins, and fats because the body is not sufficiently producing digestive enzymes anymore. Hence, the stool will not appear as oily or greasy (decrease in steatorrhea) but appear solid and formed.

You're caring for a 45 year old patient who is admitted with suspected acute pancreatitis. The patient reports having extreme mid-epigastric pain that radiates to the back. The patient states the pain started last night after eating fast food. As the nurse, you know the two most common causes of acute pancreatitis are A. High cholesterol and alcohol abuse B. History of diabetes and smoking C. Pancreatic cancer and obesity D. Gallstones and alcohol abuse

The answer is D. Main causes of acute pancreatitis are gallstones and alcohol consumption. Heavy, long-term alcohol abuse is the main cause of CHRONIC pancreatitis.

A patient is demonstrating signs and symptoms of stroke. The patient reports loss of vision. What area of the brain do you suspect is affected based on this finding? A. Brain stem B. Hippocampus C. Parietal lobe D. Occipital lobe

The answer is D. The occipital lobe is responsible for vision and color perception.

Your patient with chronic kidney disease is scheduled for dialysis in the morning. While examining the patient's telemetry strip, you note tall peaked T-waves. You notify the physician who orders a STAT basic metabolic panel (BMP). What result from the BMP confirms the EKG abnormality? A. Phosphate 3.2 mg/dL B. Calcium 9.3 mg/dL C. Magnesium 2.2 mg/dL D. Potassium 7.1 mEq/L

The answer is D. The patient's potassium level is extremely elevated. A normal potassium level is 3.5-5.1 mEq/L. This patient is experiencing hyperkalemia, which can cause tall peak T-waves. Remember in CKD (especially prior to dialysis), the patient will experience electrolyte imbalances, especially hyperkalemia.

A patient who suffered a stroke one month ago is experiencing hearing problems along with issues learning and showing emotion. On the MRI what lobe in the brain do you expect to be affected? A. Frontal lobe B. Occipital lobe C. Parietal lobe D. Temporal

The answer is D. The temporal lobe is responsible for hearing, learning, and feelings/emotions.

The kidneys are responsible for performing all the following functions EXCEPT? A. Activating Vitamin D B. Secreting Renin C. Secreting Erythropoietin D. Maintaining cortisol production

The answer is D. The adrenal glands are responsible for maintaining cortisol production not the kidneys.

A patient with acute renal injury has a GFR (glomerular filtration rate) of 40 mL/min. Which signs and symptoms below may this patient present with? Select all that apply: A. Hypervolemia B. Hypokalemia C. Increased BUN level D. Decreased Creatinine level

The answers are A and C. The glomerular filtration rate indicates how well the glomerulus is filtering the blood. A normal GFR tends to be 90 mL/min or higher. A GFR of 40 mL/min indicates that the kidney's ability to filter the blood is decreased. Therefore, the kidneys will be unable to remove waste and excessive water from the blood...hence hypervolemia and an increased BUN level will present in this patient. The patient will experience HYPERkalemia (not hypo) because the kidneys are unable to remove potassium from the blood. In addition, an INCREASED creatinine level (not decreased) will present because the kidneys cannot remove excessive waste products, such as creatinine.

Which of the following are treatment options for hyperthyroidism? Please select all that apply: A. Thyroidectomy B. Methimazole C. Liothyronine Sodium "Cytomel" D. Radioactive Iodine

The answers are A, B,and D. Liothyronine Sodium "Cytomel" is a treatment for hypothyroidism. All the other options are for hyperthyroidism.

You're educating a group of nursing students about left side brain damage. Select all the signs and symptoms noted with this type of stroke: A. Aphasia B. Denial about limitations C. Impaired math skills D. Issues with seeing on the right side E. Disoriented F. Depression and anger G. Impulsive H. Agraphia

The answers are A, C, D, F, and H. Patients who have left side brain damage will have aphasia, be AWARE of their limitations, impaired math skills, issues with seeing on the right side, no deficit in memory, depression/anger, cautious, and agraphia. All the other options are found in right side brain injury.

Your patient with cirrhosis has severe splenomegaly. As the nurse you will make it priority to monitor the patient for signs and symptoms of? Select all that apply : A. Thrombocytopenia B. Vision changes C. Increased PT/INR D. Leukopenia

The answers are A, C, and D. A patient with an enlarged spleen (splenomegaly) due to cirrhosis can experience thrombocytopenia (low platelet count), increased PT/INR (means it takes the patient a long time to stop bleeding), and leukopenia (low white blood cells). The spleen stores platelets and WBCs. An enlarged spleen can develop due to portal hypertension, which causes the platelets and WBCs to become stuck inside the spleen due to the increased pressure in the hepatic vein (hence lowering the count and the body's access to these important cells for survival).

You're educating a patient about transient ischemic attacks (TIAs). Select all the options that are incorrect about this condition: A. TIAs are caused by a temporary decrease in blood flow to the brain. B. TIAs produce signs and symptoms that can last for several weeks to months. C. A TIAs is a warning sign that an impending stroke may occur. D. TIAs don't require medical treatment.

The answers are B and D. Options A and C are CORRECT statements about TIAs. However, option B is wrong because TIAs produce signs and symptoms that can last a few minutes to hours and resolve (NOT several weeks to months). Option D is wrong be TIAs do require medical treatment.

Which patients are NOT a candidate for tissue plasminogen activator (tPA) for the treatment of stroke? A. A patient with a CT scan that is negative. B. A patient whose blood pressure is 200/110. C. A patient who is showing signs and symptoms of ischemic stroke. D. A patient who received Heparin 24 hours ago.

The answers are B and D. Patients who are experiencing signs and symptoms of a hemorrhagic stroke, who have a BP for >185/110, and has received heparin or any other anticoagulants etc. are NOT a candidate for tPA. tPA is only for an ischemic stroke.

You are receiving shift report on a patient with cirrhosis. The nurse tells you the patient's bilirubin levels are very high. Based on this, what assessment findings may you expect to find during your head-to-toe assessment? Select all that apply: A. Frothy light-colored urine B. Dark brown urine C. Yellowing of the sclera D. Dark brown stool E. Jaundice of the skin F. Bluish mucous membranes

The answers are B, C, and E. High bilirubin levels are because the hepatocytes are no longer able to properly conjugate the bilirubin because they are damaged. This causes bilirubin to leak into the blood and urine (rather than entering the bile and being excreted in the stool). Therefore, the bilirubin stays in the blood and will enter the urine. This will cause the patient to experience yellowing of the skin, sclera of the eyes, and mucous membranes ("jaundice") and have dark brown urine. The stools would be CLAY-COLORED not dark brown (remember bilirubin normally gives stool it brown color but it will be absent).

A patient has right side brain damage from a stroke. Select all the signs and symptoms that occurs with this type of stroke: A. Right side hemiplegia B. Confusion on date, time, and place C. Aphasia D. Unilateral neglect E. Aware of limitations F. Impulsive G. Short attention span H. Agraphia

The answers are B, D, F, and G. Patients who have right side brain damage will have LEFT side hemiplegia (opposite side), confused on date, time, and place, unilateral neglect (left side neglect), DENIAL about limitations, be impulsive, and have a short attention span. Agraphia, right side hemiplegia, aware of limitations, and aphasia occur in a LEFT SIDE brain injury.

A 36 year old male patient is diagnosed with acute kidney injury. The patient is voiding 4 L/day of urine. What complication can arise based on the stage of AKI this patient is in? Select all that apply: A. Water intoxication B. Hypotension C. Low urine specific gravity D. Hypokalemia E. Normal GFR

The answers are: B, C, and D. This patient is in the DIURESIS stage of AKI. The nephrons are now starting to filter out waste but cannot concentrate the urine. There is now a high amount of urea in the filtrate (because the nephrons can filter the urea out of the blood) and this causes osmotic diuresis. Urinary output will be excessive (3 to 6 L/day). Therefore, the patient is at risk for hypotension, diluted urine (low urine specific gravity), and hypokalemia (waste potassium in the urine). The patient is not at risk for water intoxication and will not have a normal GFR until the recovery stage.

Select all the patients below that are at risk for acute intra-renal injury? A. A 45 year old male with a renal calculus. B. A 65 year old male with benign prostatic hyperplasia. C. A 25 year old female receiving chemotherapy. D. A 36 year old female with renal artery stenosis. E. A 6 year old male with acute glomerulonephritis. F. An 87 year old male who is taking an aminoglycoside medication for an infection.

The answers are: C, E, and F. These patients are at risk for an intra-renal injury, which is where there is damage to the nephrons of kidney. The patients in options A and B are at risk for POST-RENAL injury because there is an obstruction that can cause back flow of urine into the kidney, which can lead to decreased function of the kidney. The patient in option D is at risk for PRE-RENAL injury because there is an issue with perfusion to the kidney.

The client with chronic obstructive pulmonary disease (COPD) is taking theophylline. The nurse should instruct the client to report which of the following signs of theophylline toxicity? Select all that apply. a. Nausea. b. Vomiting. c. Seizures. d. Insomnia. e. Vision changes.

a, b, c, d The therapeutic range for serum theophylline is 10 to 20 mcg/mL (55.5 to 111 μmol/L). At higher levels, the client will experience signs of toxicity such as nausea, vomiting, seizure, and insomnia.

Which findings will the nurse expect when caring for a patient with chronic kidney disease (CKD)? Select all that apply. a. Anemia b. Dehydration c. Hypertension d. Hypercalcemia e. Increased risk for fractures f. Elevated white blood cells

a. Anemia c. Hypertension e. Increased risk for fractures When the kidney fails, erythropoietin in not excreted, so anemia is expected. Hypocalcemia from chronic renal disease stimulates the parathyroid to release parathyroid hormone, causing calcium liberation from bones increasing the risk of pathological fracture. Dehydration and hypercalcemia are not expected in chronic renal disease. Fluid volume overload and hypocalcemia are expected. Although impaired immune function should be expected, elevated white blood cells would indicate inflammation or infection not associated with chronic renal failure itself but a complication.

A 78-yr-old patient has stage 3 CKD and is being taught about a low-potassium diet. The nurse knows the patient understands the diet when the patient selects which foods to eat? a. Apple, green beans, and a roast beef sandwich b. Granola made with dried fruits, nuts, and seeds c. Watermelon and ice cream with chocolate sauce d. Bran cereal with ½ banana and milk and orange juice

a. Apple, green beans, and a roast beef sandwich When the patient selects an apple, green beans, and a roast beef sandwich, the patient demonstrates understanding of the low-potassium diet. Granola, dried fruits, nuts and seeds, milk products, chocolate sauce, bran cereal, banana, and orange juice all have elevated levels of potassium, at or above 200 mg per 1/2 cup.

Which assessment findings would alert the nurse that the patient has entered the diuretic phase of acute kidney injury (AKI)? Select all that apply. a. Dehydration b. Hypokalemia c. Hypernatremia d. BUN increases e. Urine output increases f. Serum creatinine increases

a. Dehydration b. Hypokalemia e. Urine output increases The hallmark of entering the diuretic phase is the production of copious amounts of urine. Dehydration, hypokalemia, and hyponatremia occur in the diuretic phase of AKI because the nephrons can excrete wastes but not concentrate urine. Serum BUN and serum creatinine levels begin to decrease.

A patient on a medical unit has a potassium level of 6.8 mEq/L. What is the priority action that the nurse should take? a. Place the patient on a cardiac monitor. b. Check the patient's BP. c. Instruct the patient to avoid high-potassium foods. d. Call the lab and request a redraw of the lab to verify results.

a. Dysrhythmias may occur with an elevated potassium level and are potentially lethal. Monitor the rhythm while contacting physician or calling the rapid response team.

Heparin is ordered for a patient with a non-ST-segment-elevation myocardial infarction (NSTEMI). What is the purpose of the heparin? a. Heparin prevents the development of new clots in the coronary arteries. b. Heparin decreases coronary artery plaque size. c. Heparin enhances platelet aggregation. d. Heparin dissolves clots that are blocking blood flow in the coronary arteries.

a. Heparin prevents the development of new clots in the coronary arteries.

The patient has rapidly progressing glomerular inflammation. Weight has increased and urine output is steadily declining. What is the priority nursing intervention? a. Monitor the patient's cardiac status. b. Teach the patient about hand washing. c. Obtain a serum specimen for electrolytes. d. Increase direct observation of the patient.

a. Monitor the patient's cardiac status. The nurse's priority is to monitor the patient's cardiac status. With the rapidly progressing glomerulonephritis, renal function begins to fail and fluid, potassium, and hydrogen retention lead to hypervolemia, hyperkalemia, and metabolic acidosis. Excess fluid increases the workload of the heart, and hyperkalemia can lead to life-threatening dysrhythmias. Teaching about hand washing and observation of the patient are important nursing interventions but are not the priority. Electrolyte measurement is a collaborative intervention that will be done as ordered by the health care provider.

A plan of care for the patient with COPD could include (select all that apply) a. exercise such as walking. b. high flow rate of O2 administration. c. low-dose chronic oral corticosteroid therapy. d. use of peak flow meter to monitor the progression of COPD. e. breathing exercises such as pursed-lip breathing that focus on exhalation.

a. exercise such as walking. e. breathing exercises such as pursed-lip breathing that focus on exhalation. Breathing exercises may assist the patient during rest and activity (e.g., lifting, walking, stair climbing) by decreasing dyspnea, improving oxygenation, and slowing the respiratory rate. The main type of breathing exercise commonly taught is pursed-lip breathing. Walking or other endurance exercises (e.g., cycling) combined with strength training is probably the best intervention to strengthen muscles and improve the endurance of a patient with chronic obstructive pulmonary disease (COPD).

A college athlete is seen in the clinic 6 weeks after a concussion. Which assessment information will the nurse collect to determine whether a patient is developing postconcussion syndrome? a. Short-term memory b. Muscle coordination c. Glasgow Coma Scale d. Pupil reaction to light

a. short term memory

When admitting a 42-year-old patient with a possible brain injury after a car accident to the emergency department (ED), the nurse obtains the following information. Which finding is most important to report to the health care provider? a. The patient takes warfarin (Coumadin) daily. b. The patient's blood pressure is 162/94 mm Hg. c. The patient is unable to remember the accident. d. The patient complains of a severe dull headache.

a. the patient takes wafarin daily

A client with subdural hematoma was given mannitol to decrease intracranial pressure (ICP). Which of the following results would best show the mannitol was effective? a.) Urine output increases b.) Pupils are 8 mm and nonreactive c.) Systolic blood pressure remains at 150 mm Hg d.) BUN and creatinine levels return to normal

a.) Urine output increases Rationale: Mannitol promotes osmotic diuresis by increasing the pressure gradient in the renal tubes. Fixed and dilated pupils are symptoms of increased ICP or cranial nerve damage. No information is given about abnormal BUN and creatinine levels or that mannitol is being given for renal dysfunction or blood pressure maintenance.

The home care nurse visits a 34-yr-old woman receiving peritoneal dialysis. Which statement indicates a need for immediate follow-up by the nurse? a. "Drain time is faster if I rub my abdomen." b. "The fluid draining from the catheter is cloudy." c. "The drainage is bloody when I have my period." d. "I wash around the catheter with soap and water."

b. "The fluid draining from the catheter is cloudy." The primary clinical manifestation of peritonitis is a cloudy peritoneal effluent. Blood may be present in the effluent of women who are menstruating, and no intervention is indicated. Daily catheter care may include washing around the catheter with soap and water. Drain time may be facilitated by gently massaging the abdomen.

The nurse is teaching a patient who has a new prescription for the antithyroid drug propylthiouracil (PTU). Which statement by the nurse is correct? a. "There are no food restrictions while on this drug." b. "You need to avoid foods high in iodine, such as iodized salt, seafood, and soy products." c. "This drug is given to raise the thyroid hormone levels in your blood." d. "Take this drug in the morning on an empty stomach."

b. "You need to avoid foods high in iodine, such as iodized salt, seafood, and soy products."

A client is beginning a regimen of ferrous sulfate or iron. As you prepare to administer the medication, it is important for you to advise the client that a. Her urine will turn a dark orange b. Her bowel movements will be dark and tarry c. Her appetite will be diminished d. Her vision will become slightly blurred

b. Her bowel movements will be dark and tarry

The nurse will suspect that the patient with stable angina is experiencing a side effect of the prescribed metoprolol (Lopressor) if the a. patient is restless and agitated. b. blood pressure is 90/54 mm Hg. c. patient complains about feeling anxious. d. cardiac monitor shows a heart rate of 61 beats/minute.

b. blood pressure is 90/54 mm Hg.

A 20-year-old male patient is admitted with a head injury after a collision while playing football. After noting that the patient has developed clear nasal drainage, which action should the nurse take? a. Have the patient gently blow the nose. b. Check the drainage for glucose content. c. Teach the patient that rhinorrhea is expected after a head injury. d. Obtain a specimen of the fluid to send for culture and sensitivity.

b. check the drainage for glucose content

A 23-year-old patient who is suspected of having an epidural hematoma is admitted to the emergency department. Which action will the nurse plan to take? a. Administer IV furosemide (Lasix). b. Prepare the patient for craniotomy. c. Initiate high-dose barbiturate therapy. d. Type and crossmatch for blood transfusion.

b. prepare the patient for craniotomy

The nurse is caring for the client with increased intracranial pressure. The nurse would note which of the following trends in vital signs if the ICP is rising? a.) Increasing temperature, increasing pulse, increasing respirations, decreasing blood pressure. b.) Increasing temperature, decreasing pulse, decreasing respirations, increasing blood pressure. c.) Decreasing temperature, decreasing pulse, increasing respirations, decreasing blood pressure. d.) Decreasing temperature, increasing pulse, decreasing respirations, increasing blood pressure.

b.) Increasing temperature, decreasing pulse, decreasing respirations, increasing blood pressure. Rationale: A change in vital signs may be a late sign of increased intracranial pressure. Trends include increasing temperature and blood pressure and decreasing pulse and respirations. Respiratory irregularities also may arise.

An 83 year old female patient was found lying on the bathroom floor. She said she fell 2 days ago and has not been able to take her heart medicine or eat or drink anything since then. What conditions could be causing prerenal AKI in the patient(select all that apply)? a. anaphlyaxis b. renal calculi c. hypovolemia d. nephrotoxic drugs e. decreased cardiac output

c,e. Because the patient has had nothing to eat or drink for 2 days, she is probably dehydrated and hypovolemic. Decreased cardiac output is most likely because she is older and takes heart medicine, wich is probably for heart failure or HTN.

A 56-yr-old woman with type 2 diabetes mellitus and chronic kidney disease has a serum potassium level of 6.8 mEq/L. Which finding will the nurse monitor for? a. Fatigue b. Hypoglycemia c. Cardiac dysrhythmias d. Elevated triglycerides

c. Cardiac dysrhythmias Hyperkalemia is the most serious electrolyte disorder associated with kidney disease. Fatal dysrhythmias can occur when the serum potassium level reaches 7 to 8 mEq/L. Fatigue and hypertriglyceridemia may be present but do not require urgent intervention. Hypoglycemia is a complication related to diabetes control, not hyperkalemia. However, administration of insulin and dextrose is an emergency treatment for hyperkalemia.

During hemodialysis, the patient develops light-headedness and nausea. What should the nurse do first? a. Administer hypertonic saline. b. Administer a blood transfusion. c. Decrease the rate of fluid removal. d. Administer antiemetic medications.

c. Decrease the rate of fluid removal. The patient is experiencing hypotension from a rapid removal of vascular volume. The rate and volume of fluid removal will be decreased, and 0.9% saline solution may be infused. Hypertonic saline is not used because of the high sodium load. A blood transfusion is not indicated. Antiemetic medications may help the nausea but would not help the hypovolemia.

A patient who has chest pain is admitted to the emergency department (ED) and all of the following are ordered. Which one should the nurse arrange to be completed first? a. Chest x-ray b. Troponin level c. Electrocardiogram (ECG) d. Insertion of a peripheral IV

c. Electrocardiogram (ECG)

A patient is recovering in the intensive care unit (ICU) 24 hours after receiving a kidney transplant. What is an expected assessment finding during the earliest stage of recovery? a. Hypokalemia b. Hyponatremia c. Large urine output d. Leukocytosis with cloudy urine output

c. Large urine output Patients frequently experience diuresis in the hours and days immediately following a kidney transplant. Electrolyte imbalances and signs of infection are unexpected findings that warrant prompt intervention.

Which electrocardiographic (ECG) change is most important for the nurse to report to the health care provider when caring for a patient with chest pain? a. Inverted P wave b. Sinus tachycardia c. ST-segment elevation d. First-degree atrioventricular block

c. ST-segment elevation

A client with emphysema should receive only 1 to 3 L/minute of oxygen, if needed, or he may lose his hypoxic drive. Which of the following statements is correct about hypoxic drive? a. The client doesn't notice he needs to breathe. b. The client breathes only when his oxygen levels climb above a certain point. c. The client breathes only when his oxygen levels dip below a certain point. d. The client breathes only when his carbon dioxide level dips below a certain point.

c. The client breathes only when his oxygen levels dip below a certain point. Clients with emphysema breathe when their oxygen levels drop to a certain level; this is known as the hypoxic drive. They don't take a breath when their levels of carbon dioxide are higher than normal, as do those with healthy respiratory physiology. If too much oxygen is given, the client has little stimulus to take another breath. In the meantime, his carbon dioxide levels continue to climb, and the client will pass out, leading to a respiratory arrest.

A patient with anemia is experiencing increased fatigue and occasional palpitations at rest. The nurse would expect the patient's laboratory findings to include a. normal red blood cell (RBC) indices. b. a hematocrit (Hct) of 38%. c. a hemoglobin (Hb) of 8.6 g/dL (86 g/L). d. an RBC count of 4,500,000/L.

c. a hemoglobin (Hb) of 8.6 g/dL (86 g/L). The patient's clinical manifestations indicate moderate anemia, which is consistent with an Hb of 6 to 10 g/dL. The other values are all within the range of normal.

When monitoring the laboratory values for a patient who is taking antithyroid drugs, the nurse knows to watch for a. increased platelet counts. b. increased white blood cell counts. c. increased blood urea nitrogen level. d. increased blood glucose levels.

c. increased blood urea nitrogen level.

A 41-year-old patient who is unconscious has a nursing diagnosis of ineffective cerebral tissue perfusion related to cerebral tissue swelling. Which nursing intervention will be included in the plan of care? a. Encourage coughing and deep breathing. b. Position the patient with knees and hips flexed. c. Keep the head of the bed elevated to 30 degrees. d. Cluster nursing interventions to provide rest periods.

c. keep the head of the bed elevated to 30 degrees

The arterial blood gases of a client with chronic obstructive pulmonary disease (COPD) deteriorate, and respiratory failure is impending. For which clinical indicator should the nurse assess first? a. Cyanosis b. Bradycardia c. Mental confusion d. Distended neck veins

c. mental confusion Decreased oxygen to the vital centers in the brain results in restlessness and confusion. Cyanosis is a late sign of respiratory failure. Tachycardia, not bradycardia, will occur as a compensatory mechanism to help increase oxygen to body cells. Distended neck veins occur with fluid volume excess (e.g., pulmonary edema).

A 50-year-old woman weighs 95 kg and has a history of tobacco use, high blood pressure, high sodium intake, and sedentary lifestyle. When developing an individualized care plan for her, the nurse determines that the most important risk factors for peripheral artery disease (PAD) that need to be modified are a.weight and diet. b.activity level and diet. c.tobacco use and high blood pressure. d.sedentary lifestyle and high blood pressure.

c. tobacco use and high blood pressure Significant risk factors for peripheral artery disease include tobacco use, hyperlipidemia, elevated levels of high-sensitivity C-reactive protein, diabetes mellitus, and uncontrolled hypertension; the most important is tobacco use. Other risk factors include family history, hypertriglyceridemia, hyperuricemia, increasing age, obesity, sedentary lifestyle, and stress.

Which of the following signs and symptoms of increased ICP after head trauma would appear first? a.) Bradycardia b.) Large amounts of very dilute urine c.) Restlessness and confusion d.) Widened pulse pressure

c.) Restlessness and confusion Rationale: The earliest symptom of elevated ICP is a change in mental status. Bradycardia, widened pulse pressure, and bradypnea occur later. The client may void large amounts of very dilute urine if there's damage to the posterior pituitary.

What are intrarenal causes of AKI (select all that apply)? a. anaphylaxis b. renal stones c. bladder cancer d. nephrotoxic drugs e. acute glomerulonephritis f. tubular obstruction by myoglobin

d, e, f. Intrarenal causes of AKI includes conditions that cause direct damage to the kidney tissue, including nephrotoxic drugs, acute glomerulonephritis, and tubular obstruction by myoglobin, or prolonged ischemia.

In caring for the patient with AKI, what should the nurse be aware of? a. The most common cause of death in AKI is irreversible metabolic acidosis. b. During the oliguric phase of AKI, daily fluid intake is limited to 1000 ml plus the prior day's measured fluid loss c. dietary sodium and potassium during the oliguric phase of AKI are manage according to the patient's urinary output. d. One of the most important nursing measures in managing fluid balance in the patient with AKI in taking accurate daily weights

d. Measuring daily weights with the same scale at the same time each day allows for the evaluation and detection of excessive body fluid gains or losses.

What indicates to the nurse a patient with AKI is the recovery phase? a. a return to normal weight b. a urine output of 3700 ml/day c. decreasing sodium and potassium levels d. decreasing blood urea nitrogen and creatinine levels

d. The BUN and creatinine levels remain high during oliguric and diuretic phrases of AKI. The recovery phase begins when the glomerular filtration returns to a rate at which BUN and creatinine stabilize and then decrease. Urinary output of 305 L/day, decreasing sodium and potassium levels, and fluid weight loss are characteristic of the diuretic phase of AKI

The pharmacy has called a patient to notify her that the current brand of thyroid replacement hormone is on back order. The patient calls the clinic to ask what to do. Which is the best response by the nurse? a. "Go ahead and take the other brand that the pharmacy has available for now." b. "You can stop the medication until your current brand is available." c. "You can split the thyroid pills that you have left so that they will last longer." d. "Let me ask your prescriber what needs to be done; we will need to watch how you do if you switch brands."

d. "Let me ask your prescriber what needs to be done; we will need to watch how you do if you switch brands."

A patient has been receiving oxygen per nasal cannula while hospitalized for COPD. The patient asks the nurse whether oxygen use will be needed at home. What is the most appropriate response by the nurse? a. "Long-term home oxygen therapy should be used to prevent respiratory failure." b. "Oxygen will not be needed until or unless you are in the terminal stages of this disease." c. "Long-term home oxygen therapy should be used to prevent heart problems related to COPD." d. "You will not need oxygen until your oxygen saturation drops to 88% and you have symptoms of hypoxia."

d. "You will not need oxygen until your oxygen saturation drops to 88% and you have symptoms of hypoxia." Long-term oxygen therapy in the home will not be considered until the oxygen saturation is less than or equal to 88% and the patient has signs of tissue hypoxia, such as cor pulmonale, erythrocytosis, or impaired mental status. PaO2 less than 55 mm Hg will also allow home oxygen therapy to be considered.

A patient with a 25-year history of type 1 diabetes mellitus is reporting fatigue, edema, and an irregular heartbeat. On assessment, the nurse notes newly developed hypertension and uncontrolled blood sugars. Which diagnostic study is most indicative of chronic kidney disease (CKD)? a. Serum creatinine b. Serum potassium c. Microalbuminuria d. Calculated glomerular filtration rate (GFR)

d. Calculated glomerular filtration rate (GFR) The best study to determine kidney function or CKD that would be expected in the patient with diabetes is the calculated GFR that is obtained from the patient's age, gender, race, and serum creatinine. It would need to be abnormal for 3 months to establish a diagnosis of CKD. A creatinine clearance test done with a blood sample and a 24-hour urine collection is also important. Serum creatinine is not the best test for CKD because the level varies with different patients. Serum potassium levels could explain why the patient has an irregular heartbeat. The finding of microalbuminuria can alert the patient with diabetes about potential renal involvement and potentially failing kidneys. However, urine albumin levels are not used for diagnosis of CKD.

A patient who has had chest pain for several hours is admitted with a diagnosis of rule out acute myocardial infarction (AMI). Which laboratory test should the nurse monitor to help determine whether the patient has had an AMI? a. Myoglobin b. Homocysteine c. C-reactive protein d. Cardiac-specific troponin

d. Cardiac-specific troponin

A 52-year-old female patient was exposed to human immunodeficiency virus (HIV) 2 weeks ago through sharing needles with other substance users. What symptoms will the nurse teach the patient to report that would indicate the patient has developed an acute HIV infection? a. Cough, diarrhea, headaches, blurred vision, muscle fatigue b. Night sweats, fatigue, fever, and persistent generalized lymphadenopathy c. Oropharyngeal candidiasis or thrush, vaginal candidal infection, or oral or genital herpes d. Flu-like symptoms such as fever, sore throat, swollen lymph glands, nausea, or diarrhea

d. Flu-like symptoms such as fever, sore throat, swollen lymph glands, nausea, or diarrhea Clinical manifestations of an acute infection with HIV include flu-like symptoms between 2 to 4 weeks after exposure. Early chronic HIV infection clinical manifestations are either asymptomatic or include fatigue, headache, low-grade fever, night sweats, and persistent generalized lympadenopathy. Intermediate chronic HIV infection clinical manifestations include candidal infections, shingles, oral or genital herpes, bacterial infections, Kaposi sarcoma, or oral hairy leukoplakia. Late chronic HIV infection or acquired immunodeficiency syndrome (AIDS) includes opportunistic diseases (infections and cancer).

When caring for a client with a coagulation disorder, your primary focus should be on: a. Prevention of infection b. Pain management c. Reducing edema d. Prevention of injury and hemorrhage

d. Prevention of injury and hemorrhage

The nurse is providing postoperative care for a 30-year-old female patient after an appendectomy. The patient has tested positive for human immunodeficiency virus (HIV). What type of precautions should the nurse observe to prevent the transmission of this disease? a. Droplet precautions b. Contact precautions c. Airborne precautions d. Standard precautions

d. Standard precautions Standard precautions are indicated for prevention of transmission of HIV to the health care worker. HIV is not transmitted by casual contact or respiratory droplets. HIV may be transmitted through sexual intercourse with an infected partner, exposure to HIV-infected blood or blood products, and perinatal transmission during pregnancy, at delivery, or though breastfeeding.


संबंधित स्टडी सेट्स

Cognition E-Book Quiz Questions 1

View Set

Nursing Management: Patients With Intestinal and Rectal Disorders

View Set

Computer Security Midterm CH4, Computer Security Midterm CH 2, Information Security Exam 1, Chapter 2 Quiz, Chapter 1, CS356 CSU midterm 1

View Set

CPLP Performance Improvement: Chapter Quiz

View Set

Investment Theory, Models and Alternative Investments

View Set

A Look at the Fast-Food Industry by Eric Schlosser

View Set

The Neurological System (Part 2) Test

View Set